You are on page 1of 85

Online Prelims TEST - 23 (TEXTBOOK)

( InsightsIAS Mock Test Series for UPSC Preliminary Exam 2020 )

1 The laws that deal with the suspension of Internet services include
1. The Information Technology Act, 2000
2. The Indian Evidence Act, 1872
3. The Criminal Procedure Code (CrPC), 1973
4. The Telegraph Act, 1885

Select the correct answer using the codes below.


A. 1 and 2 only
B. 2, 3 and 4 only
C. 1, 3 and 4 only
D. 1, 2, 3 and 4

Correct Answer : C

Answer Justification :

Justification: S2: The Indian Evidence Act, originally passed in India by the Imperial Legislative
Council in 1872, during the British Raj, contains a set of rules and allied issues governing
admissibility of evidence in the Indian courts of law. It does not deal with internet shutdowns.

S1, S3 and S4: India also tops the list of Internet shutdowns globally. According to Software
Freedom Law Center’s tracker, there have been 381 shutdowns since 2012, 106 of which were in
2019.

The Information Technology Act, 2000, the Criminal Procedure Code (CrPC), 1973 and the
Telegraph Act, 1885 are the three laws that deal with suspension of Internet services.

Before 2017, Internet suspension orders were issued under section 14 of the CrPC.

In 2017, the central government notified the Temporary Suspension of Telecom Services (Public
Emergency or Public Service) Rules under the Telegraph Act to govern suspension of Internet.
These Rules derive their powers from Section 5(2) of the Indian Telegraph Act, which talks about
interception of messages in the “interests of the sovereignty and integrity of India”.

Temporary Suspension of Telecom Services [Public Emergency or Public Service] Rules, 2017:

According to these rules, only the Home Secretary of the country and a secretary of a state’s
home department can pass such an order.

These also state that any such order should be taken up by a review committee within five
days.

Despite the 2017 rules, the government has often used the broad powers under Section 144.

Learning: A recent SC judgment says the following on the rules to be followed for internet
shutdown:
Online Prelims TEST - 23 (TEXTBOOK)
( InsightsIAS Mock Test Series for UPSC Preliminary Exam 2020 )

The court recognised that the 2017 Rules are the only procedure to be followed to suspend
Internet services in the occurrence of a “public emergency” or for it to be “in the interest of
public safety”.

The verdict reiterated that the competent authority to issue an order under the Suspension
Rules, in ordinary circumstances, would be the Secretary to the Ministry of Home Affairs.

The Rules also say that in case the confirmation does not come from a competent authority,
the orders shall cease to exist within a period of 24 hours.

Clear reasons for such orders need to be given in writing, and need to be forwarded to a
Review Committee by the next working day.

The confirmation must not be a mere formality, but must indicate independent application of
mind by the competent authority to the order passed by the authorised officer, who must also
take into account changed circumstances if any, etc.

Q Source:
https://www.insightsonindia.com/2020/01/11/insights-daily-current-affairs-pib-summary-11-january-
2020/

2 Ordinarily, for certain class of minerals, the state governments grant permission for mining for the
minerals located within the boundary of the state. However, Central government approval is necessary
before granting the mineral concession for the minerals specified in the First Schedule to the Mines
and Minerals (Development and Regulation) Act, 1957; these minerals include
1. Gold
2. Iron Ore
3. Hydrocarbons

Select the correct answer using the codes below.


A. 2 and 3 only
B. 1 and 3 only
C. 1, 2 and 3
D. 1 and 2 only

Correct Answer : C

Answer Justification :

Justification: The state governments grant permission for mining, known as mineral concessions,
for all the minerals located within the boundary of the state, under the provisions of the Mines and
Minerals (Development and Regulation) Act, 1957 and Mineral Concession Rules, 1960.
Online Prelims TEST - 23 (TEXTBOOK)
( InsightsIAS Mock Test Series for UPSC Preliminary Exam 2020 )

However, for minerals specified in the First Schedule to the Mines and Minerals (Development and
Regulation) Act, 1957, Central government approval is necessary before granting the mineral
concession.

Minerals specified under the First Schedule include hydrocarbons, atomic minerals and metallic
minerals such as iron ore, bauxite copper ore, lead precious stones, zinc and gold.

Q Source:
https://www.insightsonindia.com/2020/01/10/insights-daily-current-affairs-pib-summary-10-january-
2020/

3 Consider the following statements.


1. Algae can store energy in the form of oils and carbohydrates.
2. Algae can be used for production of bio-degradable plastic.
3. Algae can be used to purify wastewater.

Select the correct answer using the codes below.


A. 2 only
B. 1 and 3 only
C. 1 and 2 only
D. 1, 2 and 3

Correct Answer : D

Answer Justification :

Justification: S1 and S2: Algae produce a variety of base materials that can be used for bio-plastics
production. Most important are carbohydrates and hydrocarbons. The algae Botryococcus braunii
has the capacity to produce and excrete these materials into the medium.

Microalgae contain similar raw materials as traditional crops: high-quality oils, proteins, pigments
as well as hydrocarbons and sugars. Culturing microalgae has several advantages over conventional
farming:

algae don’t need agricultural land, therefore there is no competition for food or farmland

microalgae have much higher yields per hectare and are extremely efficient with water

algae may grow on nutrients from residual streams, like waste water and CO2.

S3: In other words, Algae thrive in nutrient-rich waters like municipal waste waters (sewage),
animal wastes and some industrial effluents, at the same time purifying these wastes while
producing a biomass suitable for biofuels production.
Online Prelims TEST - 23 (TEXTBOOK)
( InsightsIAS Mock Test Series for UPSC Preliminary Exam 2020 )

Algae store energy in the form of oils and carbohydrates, which, combined with their high
productivity, means they can produce from 2,000 to as many as 5,000 gallons of biofuels per acre
per year.

Also see
https://www.wur.nl/en/show/Development-of-bioplastic-production-technologies-from-microalgae.ht
m

Q Source: ICSE Environmental Science: Xth

4 Which of the following personalities is related to Carnatic music?


1. Tyagaraja
2. Muthuswami Dikshitar
3. Syama Sastri

Select the correct answer using the codes below.


A. 2 only
B. 1 and 3 only
C. 1 and 2 only
D. 1, 2 and 3

Your Answer : D
Correct Answer : D

Answer Justification :

Learning: The birth of the Musical Trinity - Tyagaraja, Muthuswami Dikshitar and Syama Sastri - at
Tiruvarur between the years 1750 to 1850 A.D. ushered in an era of dynamic development in
Carnatic music.

The Trinity were not only contemporaries among themselves but, also contemporaries of great
composers of Western Music, as Beethoven, Mozart, Wagner and Haydn.

It was the 'Golden Age' of music throughout the world. Carnatic music reached its pinnacle of
artistic excellence during this period.

A galaxy of composers in the post-Trinity period kept the banner of Carnatic music flying high
such as Vina Kuppayyar, Patnam Subrahmanya Iyer, Ramnad Srinivasa Iyengar, Mysore
Sadasiva Rao, Mysore Vasudevathar and Papanasam Sivan.

Q Source: http://www.ccrtindia.gov.in/performingart.php

5 Consider the following statements.


1. Commercial exploitation of forest produce is not allowed in national parks.
Online Prelims TEST - 23 (TEXTBOOK)
( InsightsIAS Mock Test Series for UPSC Preliminary Exam 2020 )

2. No grazing or private tenurial rights, land rights are allowed in National Parks.

Select the correct answer using the codes below.


A. 1 only
B. 2 only
C. Both 1 and 2
D. None of the above

Your Answer : C
Correct Answer : C

Answer Justification :

Justification: The Wildlife (Protection) Act, 1972 (Last amended in 2006) provides a powerful legal
framework for:

Prohibition of hunting

Protection and management of wildlife habitats

Establishment of protected areas

Regulation and control of trade in parts and products derived from wildlife

Management of zoos.

National parks and Tiger Reserves are by law more strictly protected, allowing virtually no human
activity except that which is in the interest of wildlife conservation. Grazing and private tenurial
rights are disallowed in National Parks but can be allowed in sanctuaries at the discretion of the
Chief Wildlife Warden.

The amended WLPA does not allow for any commercial exploitation of forest produce in both
national parks and wildlife sanctuaries, and local communities can collect forest produce only for
their bona fide needs.
th
Q Source: AR: ICSE 10 Environment Science

6 Consider the following statements.


Assertion (A): Occurrence of typhoons is a chief feature associated with the China type of climate.
Reason (R): The formation of tropical cyclones is strongly influenced by the temperature of the
underlying ocean and the China type of climate is characterized by year round warm oceans.

In the context of the above, which of these is correct?


A. A is correct, and R is an appropriate explanation of A.
Online Prelims TEST - 23 (TEXTBOOK)
( InsightsIAS Mock Test Series for UPSC Preliminary Exam 2020 )

B. A is correct, but R is not an appropriate explanation of A.


C. A is correct, but R is incorrect.
D. Both A and R are incorrect.

Your Answer : A
Correct Answer : A

Answer Justification :

Justification: The formation of tropical cyclones is strongly influenced by the temperature of the
underlying ocean or, more specifically, by the thermal energy available

The oceans provide the source of energy for tropical cyclones both by direct heat transfer
from their surface (known as sensible heat) and by the evaporation of water.

Occurrence of typhoons—intense tropical cyclones that originate in the Pacific Ocean, and
move westwards to the coastlands bordering the South China Sea is a chief feature associated
with the China type of climate (warm temperature eastern) due to its year round warm
temperatures.

They are most frequent in late summer, from July to September and can be very disastrous.

Q Source: Chapter 21: Goh Cheng Leong: Certificate Physical and Human Geography

7 Women, Business and the Law (WBL) is a World Bank Group project collecting unique data on the laws
and regulations that restrict women’s economic opportunities. Which of following form the
components of WBL index?
1. Mobility
2. Marriage
3. Entrepreneurship
4. Assets

Select the correct answer using the codes below.


A. 1, 2 and 3 only
B. 2 and 3 only
C. 1, 2, 3 and 4
D. 1 and 4 only

Your Answer : C
Correct Answer : C

Answer Justification :

Justification: Women, Business and the Law 2020, the sixth edition in a series, has been released.
Online Prelims TEST - 23 (TEXTBOOK)
( InsightsIAS Mock Test Series for UPSC Preliminary Exam 2020 )

The index analyzes laws and regulations affecting women’s economic inclusion in 190 economies.

It is composed by eight indicators structured around women’s interactions with the law as they
begin, progress through and end their careers, aligns different areas of the law with the economic
decisions women make at various stages of their lives.

The indicators are:

Mobility, Workplace, Pay, Marriage, Parenthood, Entrepreneurship, Assets, and Pension.

How are countries ranked?

The study tracked “how laws affect women at different stages in their working lives and
focusing on those laws applicable in the main business city”.

The Index is based on the countries’ formal laws and regulations that have a bearing on
women’s economic participation.

Q Source:
https://www.insightsonindia.com/2020/01/17/insights-daily-current-affairs-pib-summary-17-january-
2020/

8 Which of the following reason(s) makes Kathak classical dance unique and different from other
classical dances?
1. Kathak is the only form of classical dance wedded to Hindustani or the North Indian music.
2. Being the only classical dance of India having links with Muslim culture, it represents a unique
synthesis of Hindu and Muslim genius in art.

Select the correct answer using the codes below.


A. 1 only
B. 2 only
C. Both 1 and 2
D. None of the above

Your Answer : C
Correct Answer : C

Answer Justification :

Justification: The emergence of Raslila , mainly in the Braj region (Mathura in Western U.P.) was
an important development for Kathak.

It combined in itself music, dance and the narrative. Dance in Raslila, however, was mainly an
extension of the basic mime and gestures of the Kathakars or story-tellers which blended easily with
Online Prelims TEST - 23 (TEXTBOOK)
( InsightsIAS Mock Test Series for UPSC Preliminary Exam 2020 )

the existing traditional dance.

With the coming of the Mughals, this dance form received a new impetus. A transition from the
temple courtyard to the palace durbar took place which necessitated changes in presentation. In
both Hindu and Muslim courts, Kathak became highly stylised and came to be regarded as a
sophisticated form of entertainment. Under the Muslims there was a greater stress on nritya and
bhavagiving the dance graceful, expressive and sensuous dimensions.

The nineteenth century saw the golden age of Kathak under the patronage of Wajid Ali Shah, the
last Nawab of Oudh. He established the Lucknow gharana with its strong accent on bhava, the
expression of moods and emotions. The Jaipur gharana known for its layakari or rhythmic virtuosity
and the Benaras gharana are other prominent schools of Kathak dance. The technique of movement
in Kathak is unique to it.

Today, Kathak has emerged as a distinct dance form. Being the only classical dance of India having
links with Muslim culture, it represents a unique synthesis of Hindu and Muslim genius in art.
Further, Kathak is the only form of classical dance wedded to Hindustani or the North Indian music.
Both of them have had a parallel growth, each feeding and sustaining the other.

Q Source: http://ccrtindia.gov.in/kathak.php

9 Consider the following statements about Coronaviruses (CoV).


1. Coronaviruses are zoonotic, meaning they are transmitted between animals and people.
2. Human coronaviruses spread from an infected person to others only through close personal contact.
3. Middle East Respiratory Syndrome (MERS-CoV) and Severe Acute Respiratory Syndrome (SARS-
CoV) are caused by CoV.

Select the correct answer using the codes below.


A. 1 and 2 only
B. 3 only
C. 1, 2 and 3
D. 1 and 3 only

Your Answer : D
Correct Answer : D

Answer Justification :

Justification: Coronaviruses (CoV) are a large family of viruses that cause illness ranging from the
common cold to more severe diseases such as Middle East Respiratory Syndrome (MERS-CoV) and
Severe Acute Respiratory Syndrome (SARS-CoV).

Coronaviruses are zoonotic, meaning they are transmitted between animals and people.

Common signs of infection include respiratory symptoms, fever, cough, shortness of breath and
breathing difficulties. In more severe cases, infection can cause pneumonia, severe acute
respiratory syndrome, kidney failure and even death.
Online Prelims TEST - 23 (TEXTBOOK)
( InsightsIAS Mock Test Series for UPSC Preliminary Exam 2020 )

Transmission:

Human coronaviruses most commonly spread from an infected person to others through:

the air by coughing and sneezing.

close personal contact, such as touching or shaking hands.

touching an object or surface with the virus on it, then touching your mouth, nose, or eyes
before washing your hands.

rarely, fecal contamination.

Q Source:
https://www.insightsonindia.com/2020/01/18/insights-daily-current-affairs-pib-summary-18-january-
2020/

10 Which of the following is/are common between Dhrupad and Khyal genre of Indian classical music?
1. Both come from the court of Amir Khusrou of the 13th century.
2. Both are associated with the extensive use of the Been and Pakhawaj musical instruments.

Select the correct answer using the codes below.


A. 1 only
B. 2 only
C. Both 1 and 2
D. None of the above

Your Answer : A
Correct Answer : D

Answer Justification :

Justification: Both statements are wrong. The description of the two styles below will clarify the
difference.

“ Dhrupad is believed to have been a further elaboration of the prabandha structure. While it might
have had an impetus for popularity even by the 14th century, it finds a blossoming period from 15th
century onwards to about the 18th century. During these centuries we meet the most respected and
renowned singers and patrons of this form. There was Man Singh Tomar, the Maharaja of Gwalior.
It was he who was mainly responsible for the enormous vogue of dhrupad.

The dhrupad is even now highly respected and can be heard on the concert platform but more often
in temples of North India. Thedhrupad has somewhat receded to the background and is not so
popular with the masses. The Been and Pakhawaj which were closely associated with the dhrupad
Online Prelims TEST - 23 (TEXTBOOK)
( InsightsIAS Mock Test Series for UPSC Preliminary Exam 2020 )

also do not find much patronage these days.

Today the pride of place in classical Hindustani Music is occupied by the Khyal. We are really not
sure about the beginning of the Khyal. The word is alien and means 'imagination'. And as you will
find when you hear it is more lyrical than the dhrupad. But whether the musical form itself is
foreign. is a matter of doubt.

Some scholars are of the opinion that in fact, it has its roots in the ancient Indian roopaka alaps. It
is also said that Amir Khusrou of the 13th century gave it an impetus. Sultan Mohammed Sharkhi of
the 15th century is credited with encouraging this form. However, it attained its maturity at the
hands of Niyamat Khan Sadarang and Adarang of the 18th century.”

Q Source: http://ccrtindia.gov.in/hindustaniclassicalmusic.php

11 The Union Ministry of Health and Family Welfare has published a national policy for the treatment of
450 ‘rare diseases’. Consider the following about it.
1. The policy intends to kickstart a registry of rare diseases to be maintained by the NITI Aayog.
2. Genetic diseases have been kept out of the purview of the policy to exclude developmental defects.
3. A one-time curative treatment for patients suffering from rare diseases will be limited to the
beneficiaries of Pradhan Mantri Jan Arogya Yojana.

Select the correct answer using the codes below.


A. 1, 2 and 3
B. 1 only
C. 3 only
D. None of the above

Your Answer : C
Correct Answer : C

Answer Justification :

Justification: The Centre first prepared such a policy in 2017 and appointed a committee in 2018
to review it.

S1: Among other measures, the policy intends to kickstart a registry of rare diseases, which will be
maintained by the Indian Council of Medical Research (ICMR).

S2: According to the policy, rare diseases include genetic diseases, rare cancers, infectious tropical
diseases, and degenerative diseases. Most rare diseases are genetic, and are present throughout a
person’s entire life, even if symptoms do not immediately appear.

S3: Under the policy, there are three categories of rare diseases — requiring one-time curative
treatment, diseases that require long-term treatment but where the cost is low, and those needing
long-term treatments with high cost. Some of the diseases in the first category include osteopetrosis
and immune deficiency disorders, among others.

As per the policy, the assistance of Rs 15 lakh will be provided to patients suffering from rare
Online Prelims TEST - 23 (TEXTBOOK)
( InsightsIAS Mock Test Series for UPSC Preliminary Exam 2020 )

diseases that require a one-time curative treatment under the Rashtriya Arogya Nidhi scheme. The
treatment will be limited to the beneficiaries of Pradhan Mantri Jan Arogya Yojana.

Background: The policy was created on the direction of the Delhi High Court to the Ministry of
Health and Family Welfare. This was in response to writ petitions for free treatment of such
diseases, due to their “prohibitively” high cost of treatment. Hence, a policy was deemed necessary
to devise a “multipronged” and “multisectoral” approach to build India’s capacity for tackling such
ailments, including by gathering epidemiological data, arriving at a definition and estimating the
cost of such diseases.

Q Source:
https://www.insightsonindia.com/2020/01/15/insights-daily-current-affairs-pib-summary-15-january-
2020/

12 The folk music and dances of agricultural communities celebrate the rhythms of daily life and dance-
drama or folk theatre showcases the beliefs of a community. Match some of these folk elements with
the state they are celebrated in:
1. Lavani: Karnataka
2. Yakshagana: West Bengal
3. Jatra: Rajasthan

Select the correct answer using the codes below.


A. 2 only
B. 1 and 3 only
C. 1, 2 and 3
D. None of the above

Your Answer : D
Correct Answer : D

Answer Justification :

Justification: Along the entire Himalayan region, from Kashmir to Darjeeling, folk dancers link
arms and sway gracefully in undulating movements, celebrate the sowing of the wheat crop; few
can resist the infectious beat of the dholak, the two-sided drum, and pairs of dancers take turns to
execute complex acrobatic movements in the centre of a circle of abandoned dancers. Women
perform the Giddha, also characterised by its spontaneous energy. Rajasthani women, their faces
covered with flowing veils, are swirls of colour as they pirouette in the Ghoomar dance, while their
counterparts in Gujarat perform the famous Garba, dancing in a circle with batons.

Their men perform the Dandiya Ras, a more vigorous version of the same dance, leaping and
crouching in twirling patterns. In the fishing communities of Maharashtra, men and women link
arms and dance together and the women climb on to the men's shoulders to form pyramids. The
women's Lavani dance from this area is notable for its unabashed sensuality.

There are also several forms of dance-drama or folk theatre, such as the Nautanki of Rajasthan,
Uttar Pradesh and Bihar, the Bhavai of Gujarat, the irreverent Tamasha of Maharashtra , the
Bengali Jatra, the spectacular Yakshagana of Karnataka and Theyyam of Kerala, all of which
Online Prelims TEST - 23 (TEXTBOOK)
( InsightsIAS Mock Test Series for UPSC Preliminary Exam 2020 )

narrate legends of local heroes, kings and deities. Martial art forms throughout the country have
been stylized to quasi dance forms, notable among which are the martial dances of the North-
eastern hill tribes, the Lazim dances of Maharashtra, the Kalaripayattu of Kerala, and the highly
stylized masked Chhau dances of Orissa, West Bengal and Bihar.

Q Source: http://ccrtindia.gov.in/performingart.php

13 Which of the following conditions is/are suitable for the growth of seaweed?
1. Turbid water
2. Freshwater inflows
3. Water temperature ranging from 25 °C to 30 °C

Select the correct answer using the codes below.


A. 2 only
B. 1 and 3 only
C. 3 only
D. None of the above

Your Answer : C
Correct Answer : C

Answer Justification :

Justification: A water temperature ranging from 25 °C to 30 °C is best for growing seaweed. In


shallow waters near the beach, the water temperature can become quite high especially during a
sunny day. Such an area is not suitable for planting seaweed.

Fresh water kills seaweed. But, Seaweed prefers clear saline water and plenty of sunlight, even
turbid water (muddy) will not sustain good seaweed growth.

If the water current is too strong, it can damage your plants and even wash away your planted lines
by pulling down the stakes.

They are the only group of higher plants adapted to life in the salt water. Major Sea grass meadows
in India occur along the south east coast of Tamil Nadu, in the lagoons of a few Lakshadweep Island
and Andaman and Nicobar islands.

Q Source: ICSE Environmental Science: Xth

14 Consider the following statements about National Investigation Agency (NIA) Act and the NIA
Amendment Act, 2019.
1. It gives the NIA powers to take suo motu cognisance of terror activities in any part of India and
register a case.
2. It allows the NIA to enter any state without permission from the state government, and to
investigate and arrest people.

Select the correct answer using the codes below.


Online Prelims TEST - 23 (TEXTBOOK)
( InsightsIAS Mock Test Series for UPSC Preliminary Exam 2020 )

A. 1 only
B. 2 only
C. Both 1 and 2
D. None of the above

Your Answer : C
Correct Answer : C

Answer Justification :

Justification: Statement 1: The NIA Act governs the functioning of India’s premier counter-terror
agency. It was passed in the wake of the 26/11 Mumbai terrorist attacks.

It makes the National Investigation Agency the only truly federal agency in the country, along the
lines of the FBI in the United States, more powerful than the CBI.

It gives the NIA powers to take suo motu cognisance of terror activities in any part of India and
register a case, to enter any state without permission from the state government, and to investigate
and arrest people.

The 2019 NIA Amendment Act:

It expanded the type of offences that the investigative body could investigate and prosecute.
The agency can now investigate offences related to human trafficking, counterfeit currency,
manufacture or sale of prohibited arms, cyber-terrorism, and offences under the Explosive
Substances Act, 1908.

The amendment also enables the central government to designate sessions courts as special
courts for NIA trials.

It also allows an NIA officer to conduct raids, and seize properties that are suspected to be
linked to terrorist activities without taking prior permission of the Director General of Police
of a state. The investigating officer only requires sanction from the Director General of NIA.

Learning: Objections bJustification: Statement 1: The NIA Act governs the functioning of India’s
premier counter-terror agency. It was passed in the wake of the 26/11 Mumbai terrorist attacks.

It makes the National Investigation Agency the only truly federal agency in the country, along the
lines of the FBI in the United States, more powerful than the CBI.

It gives the NIA powers to take suo motu cognisance of terror activities in any part of India and
register a case, to enter any state without permission from the state government, and to investigate
and arrest people.

The 2019 NIA Amendment Act:


Online Prelims TEST - 23 (TEXTBOOK)
( InsightsIAS Mock Test Series for UPSC Preliminary Exam 2020 )

It expanded the type of offences that the investigative body could investigate and prosecute.
The agency can now investigate offences related to human trafficking, counterfeit currency,
manufacture or sale of prohibited arms, cyber-terrorism, and offences under the Explosive
Substances Act, 1908.

The amendment also enables the central government to designate sessions courts as special
courts for NIA trials.

It also allows an NIA officer to conduct raids, and seize properties that are suspected to be
linked to terrorist activities without taking prior permission of the Director General of Police
of a state. The investigating officer only requires sanction from the Director General of NIA.

Learning: Objections by Chhattisgarh:

In its petition, the Chhattisgarh government said the Act “ultra vires the Constitution” and “beyond
the legislative competence of the Parliament”.

According to the state, the 2008 Act allows the Centre to create an agency for investigation, which
is a function of the state police. ‘Police’ is an entry in the State List of the Constitution’s 7th
Schedule.

The Act takes away the state’s power of conducting an investigation through the police, while
conferring “unfettered, discretionary and arbitrary powers” on the Centre.

Q Source:
https://www.insightsonindia.com/2020/01/17/insights-daily-current-affairs-pib-summary-17-january-
2020/

15 Consider the following statements about seafloor hydrothermal systems.


1. Metal sulfides and several noble metals can be recovered from such systems.
2. These vent systems are amongst some of the most productive ecosystems on earth.

Which of the above is/are correct?


A. 1 only
B. 2 only
C. Both 1 and 2
D. None

Your Answer : A
Correct Answer : C

Answer Justification :

Justification: The discovery of some modern hydrothermal systems in the deep oceanic realm
along the mid-ocean ridges has kindled a lot of interest primarily on account of the high
concentration of base metals (Cu and Zn) and many noble metals (Au, Ag, Pd, Pt) in them.
Online Prelims TEST - 23 (TEXTBOOK)
( InsightsIAS Mock Test Series for UPSC Preliminary Exam 2020 )

Apart from their economic potential, sea-floor hydrothermal vents are characterized by dense
biologic communities.

Over 700 species have been reported, making the vents among the most productive ecosystems on
Earth.

Learning: The basic paradigm of hydrothermal activity involves sub-seafloor convective circulation
of seawater through permeable rocks mainly driven by the upper mantle heat sources.

At water depths up to 3,700 metres, hydrothermal fluids, having seeped from the ocean into
subterranean chambers where they are heated by the molten rock beneath the crust, are
discharged from the black smokers at temperatures up to 400° C.

As these fluids mix with the cold surrounding seawater, metal sulphides in the water are
precipitated onto the chimneys and nearby seabed.

These sulphides, including galena (lead), sphalerite (zinc) and chalcopyrite (copper),
accumulate at and just below the seafloor, where they form massive deposits that can range
from several thousands to about 100 million tonnes.

Q Source: Based on past year UPSC papers

http://www.ncaor.gov.in/pages/researchview/6

16 Match the following regions and their traditional water harvesting systems.
1. Korambu: Eastern Ghats
2. Kul and Naula: Western Himalayan region
3. Zabo: Kashmir

Select the correct answer using the codes below.


A. 1 and 2 only
B. 3 only
C. 1 and 3 only
D. 2 and 3 only

Your Answer : C
Correct Answer : A

Answer Justification :

Justification: In respect of availability and non-availability of water, India can be divided into 15
Ecological regions, ranging from dry, cold desert or Ladakh to the dry hot desert of Rajasthan, from
the sub-temperate mountain of the Himalayas to the tropical high mountain of Nilgiri.
Online Prelims TEST - 23 (TEXTBOOK)
( InsightsIAS Mock Test Series for UPSC Preliminary Exam 2020 )

Learning: ZING SYSTEM

Melting glaciers and snows are the only sources of water for the people residing in Ladakh region.
The people of this region make intelligent use of their limited resources and make agriculture
possible in this dry and barren land.

The snow and ice melt slowly through the day and water is available in the streams only in the
evening, when it is too late for irrigation.

The water in the streams is hence led through channels to storage tanks and used the next day.
These Storage tanks are called as zing.

APTANI SYSTEM

This system is practiced in Arunachal Pradesh by Aptani tribes. Under this system the stream water
is blocked by constructing a wall 2 to 4 m high and 1 m thick near forested hill slopes. This water is
taken to the agricultural fields through channels. The valleys are terraced into plots separated by
0.6 m high earthen darns with inlet and outlet channels (to the next plot) that help to flood or drain
the plots as when required.

ZABO SYSTEM

This traditional system is practiced in Nagaland. The term labo means 'impounding run-off'. labo is
Online Prelims TEST - 23 (TEXTBOOK)
( InsightsIAS Mock Test Series for UPSC Preliminary Exam 2020 )

practiced in Nagaland. When rain falls on terraced hill slopes, the run-off collects in ponds in the
middle terrace. The runoff then passes through slopes where there are cattle yards, and finally
reaches the paddy fields at the foot of the hills. Thus it is through this system that only the
irrigation of paddy fields is brought about, but the fertility of the crop- field is also improved.

BAMBOO DRIP IRRIGATION TRICKLE DRIP IRRIGATION

Under this system the rapidly flowing water front streams and springs is captured by bamboo pipes
and transported over hundreds of metres to drip irrigate black pepper cultivation in Meghalaya.

Many bamboo pipes of varying diameters and lengths are laid to manipulate and control the flow of
water through this system of water conservation.

Q Source: ICSE Environment: Class X

17 H9N2 viruses are associated with

A. Foot and Mouth Disease


B. Influenza epidemics
C. Widespread tuberculosis
D. Diarrhoea

Your Answer : B
Correct Answer : B

Answer Justification :

Justification: Indian scientists have detected the country’s first case of infection with a rare
variant of the virus- H9N2- that causes avian influenza, or bird flu.

H9N2 is a subtype of the influenza A virus, which causes human influenza as well as bird flu.

The H9N2 subtype was isolated for the first time in Wisconsin, US in 1966 from turkey flocks.

H9N2 viruses are found worldwide in wild birds and are endemic in poultry in many areas.

H9N2 viruses could potentially play a major role in the emergence of the next influenza
pandemic.

According to the World Health Organization (WHO), with avian influenza viruses circulating
in poultry, there is a risk for sporadic infection and small clusters of human cases due to
exposure to infected poultry or contaminated environments. Therefore, sporadic human cases
are not unexpected.

Q Source:
https://www.insightsonindia.com/2020/01/13/insights-daily-current-affairs-pib-summary-13-january-
Online Prelims TEST - 23 (TEXTBOOK)
( InsightsIAS Mock Test Series for UPSC Preliminary Exam 2020 )

2020/

18 Consider the following about “No Development Zones (NDZs)” as defined under the revised Coastal
Regulation Zone (CRZ) rules.
1. Entire islands in backwater regions are notified as NDZs due to their rich and vulnerable ecology.
2. No temporary tourism related facilities are allowed in any NDZ.

Select the correct answer using the codes below.


A. 1 only
B. 2 only
C. Both 1 and 2
D. None of the above

Your Answer : D
Correct Answer : D

Answer Justification :

Background: In India, the Coastal Regulation Zone (CRZ) Rules govern human and industrial
activity close to the coastline, in order to protect the fragile ecosystems near the sea. They restrict
certain kinds of activities — like large constructions, setting up of new industries, storage or
disposal of hazardous material, mining, reclamation and bunding — within a certain distance from
the coastline.

In 2018, fresh Rules were issued, which aimed to remove certain restrictions on building,
streamlined the clearance process, and aimed to encourage tourism in coastal areas.

In all Rules, the regulation zone has been defined as the area up to 500 m from the high-tide line.

The restrictions depend on criteria such as the population of the area, the ecological sensitivity, the
distance from the shore, and whether the area had been designated as a natural park or wildlife
zone.

Justification: Statement 2: Temporary tourism facilities such as shacks, toilet blocks, change
rooms, drinking water facilities etc. have now been permitted in Beaches. Such temporary tourism
facilities are also now permissible in the "No Development Zone" (NDZ) of the CRZ-III areas as per
the Notification. However, a minimum distance of 10 m from HTL should be maintained for setting
up of such facilities.

Statement 1: A No Development Zone (NDZ) of 20 meters has been stipulated for all Islands: For
islands close to the main land coast and for all Backwater Islands in the main land, in wake of space
limitations and unique geography of such regions, bringing uniformity in treatment of such regions,
NDZ of 20 m has been stipulated.

Learning: Densely populated rural areas to be afforded greater opportunity for development: For
CRZ-III (Rural) areas, two separate categories have now been stipulated as below:

(a) CRZ-III A - These are densely populated rural areas with a population density of 2161 per square
Online Prelims TEST - 23 (TEXTBOOK)
( InsightsIAS Mock Test Series for UPSC Preliminary Exam 2020 )

kilometre as per 2011 Census. Such areas shall have a No Development Zone (NDZ) of 50 meters
from the HTL as against 200 meters from the High Tide Line stipulated in the CRZ Notification,
2011 since such areas have similar characteristics as urban areas.

(b) CRZ-III B - Rural areas with population density of below 2161 per square kilometre as per 2011
Census. Such areas shall continue to have an NDZ of 200 meters from the HTL.

Q Source:
https://www.insightsonindia.com/2020/01/13/insights-daily-current-affairs-pib-summary-13-january-
2020/

19 Which of the following minerals may be found in your perspiration?


1. Sodium
2. Calcium
3. Zinc
4. Copper

Select the correct answer using the codes below.


A. 1 only
B. 2 and 3 only
C. 1, 2 and 4 only
D. 1, 2, 3 and 4

Your Answer : C
Correct Answer : D

Answer Justification :

Justification: Perspiration contains many minerals such as sodium (Na), potassium (K), calcium
(Ca), and magnesium (Mg). Others such as zinc, copper, iron are found in trace amounts.

Some reports suggest that loss of minerals such as copper, zinc and iron through sweat can be
substantial.

For example, it has been reported that during a marathon, iron loss can be up to 2.5 mg per liter of
sweat (a marathoner can lose 4L of sweat in one race). Adults typically need 8 (male) to 18 mg
(female) of iron per day.

Q Source: Qs based on UPSC past year papers

20 With regards to Zero Budget Natural Farming (ZBNF), consider the following statements.
1. It is a natural farming technique that uses biological pesticides instead of chemical-based fertilizers.
2. It reduces farmers’ costs through eliminating external inputs and using in-situ resources to
rejuvenate soils.
3. It does not require any electricity.

Select the correct answer using the codes below.


Online Prelims TEST - 23 (TEXTBOOK)
( InsightsIAS Mock Test Series for UPSC Preliminary Exam 2020 )

A. 1 and 2 only
B. 2 and 3 only
C. 1, 2 and 3
D. 1 and 3 only

Your Answer : A
Correct Answer : A

Answer Justification :

Justification: Statement 1: It is a method of farming where the cost of growing and harvesting
plants is zero.

This means that farmers need not purchase fertilizers and pesticides in order to ensure the healthy
growth of crops.

It is, basically, a natural farming technique that uses biological pesticides instead of chemical-based
fertilizers.

Farmers use earthworms, cow dung, urine, plants, human excreta and such biological fertilizers for
crop protection. It reduces farmers’ investment. It also protects the soil from degradation.

Statement 2: It reduces farmers’ costs through eliminating external inputs and using in-situ
resources to rejuvenate soils, whilst simultaneously increasing incomes, and restoring ecosystem
health through diverse, multi-layered cropping systems.

For instance, Cow dung from local cows has proven to be a miraculous cure to revive the fertility
and nutrient value of soil. One gram of cow dung is believed to have anywhere between 300 to 500
crore beneficial micro-organisms. These micro-organisms decompose the dried biomass on the soil
and convert it into ready-to-use nutrients for plants.

Statement 3: It may still require electricity for pumping water, tilling the soil etc. Generally
speaking, it is estimated that ZBNF requires only 10 per cent water and 10 per cent electricity than
what is required under chemical and organic farming. ZBNF may improve the potential of crops to
adapt to and be produced for evolving climatic conditions.

The Andhra Pradesh government has signed a Memorandum of Understanding (MoU) with German
firm KfW regarding Zero Budget Natural Farming (ZBNF).

Q Source:
https://www.insightsonindia.com/2020/01/10/insights-daily-current-affairs-pib-summary-10-january-
2020/

21 Consider the following statements about Harmonised System coding and HSN codes as used in India.
1. HSN classification is widely used for taxation purposes by helping to identify the rate of tax
applicable to a specific product in a country that is under review.
2. Indian government has decided not to allow imports of goods that do not bear a HSN code.

Select the correct answer using the codes below.


Online Prelims TEST - 23 (TEXTBOOK)
( InsightsIAS Mock Test Series for UPSC Preliminary Exam 2020 )

A. 1 only
B. 2 only
C. Both 1 and 2
D. None of the above

Your Answer : B
Correct Answer : C

Answer Justification :

Justification: It is a six-digit identification code. Of the six digits, the first two denote the HS
Chapter, the next two give the HS heading, and the last two give the HS subheading. HS Code is
also known as HSN Code in India. Goods are classified into Harmonized System of Nomenclature or
HSN. It is used up to 8 digit level.

These are developed by the World Customs Organization (WCO) and called the “universal economic
language” for goods. It is a multipurpose international product nomenclature.

The system currently comprises of around 5,000 commodity groups.

HSN classification is widely used for taxation purposes by helping to identify the rate of tax
applicable to a specific product in a country that is under review. It can also be used in calculations
that involve claiming benefits.

India will not allow imports without HSN code. This will enable India’s exports to be accepted
globally due to the quality of goods and services.

Learning: HS code are used by Customs authorities, statistical agencies, and other government
regulatory bodies, to monitor and control the import and export of commodities through:

Customs tariffs

Collection of international trade statistics

Rules of origin

Collection of internal taxes

Trade negotiations (e.g., the World Trade Organization schedules of tariff concessions)

Transport tariffs and statistics

Monitoring of controlled goods (e.g., wastes, narcotics, chemical weapons, ozone layer
depleting substances, endangered species, wildlife trade)
Online Prelims TEST - 23 (TEXTBOOK)
( InsightsIAS Mock Test Series for UPSC Preliminary Exam 2020 )

Areas of Customs controls and procedures, including risk assessment, information technology
and compliance.

Q Source:
https://www.insightsonindia.com/2020/01/16/insights-daily-current-affairs-pib-summary-16-january-
2020/

22 Consider the following about the classical dance styles as recognized in the country.
1. All of them have roots either in Southern or North-eastern India.
2. All of them adhere to the canons of classical dance laid down in the Natya Shastra, a second century
C.E. text ascribed to the sage Bharata.
3. All of them include music and recitation in local language or Sanskrit.

Select the correct answer using the codes below.


A. 1 only
B. 2 and 3 only
C. 1, 2 and 3
D. 2 only

Your Answer : B
Correct Answer : B

Answer Justification :

Justification: The number of recognized classical dances range from eight to more, depending on
the source and scholar. The Sangeet Natak Akademi recognizes eight – Bharatanatyam, Kathak,
Kuchipudi, Odissi, Kathakali, Sattriya, Manipuri and Mohiniyattam.

Scholars such as Drid Williams add Chhau, Yakshagana and Bhagavata Mela to the list. The Culture
Ministry of the Government of India includes Chhau in its classical list.

Statement 3: These dances are traditionally regional, all of them include music and recitation in
local language or Sanskrit, and they represent a unity of core ideas in a diversity of styles, costumes
and expression.

Statement 1: Odissi and Kathak have roots in Odisha, eastern India. Manipuri and Sattriya from NE
India, and the rest from Southern India.

Statement 2: In their present format, their history cannot be traced back to over two to three
hundred years, but they all have links with the ancient and medieval literary, sculptural and musical
traditions of India and of their particular regions. They all adhere to the canons of classical dance
laid down in the Natya Shastra, a second century C.E. text ascribed to the sage Bharata, to whom it
was supposedly revealed by the Creator, Brahma.

Q Source: http://ccrtindia.gov.in/performingart.php

https://en.wikipedia.org/wiki/Indian_classical_dance
Online Prelims TEST - 23 (TEXTBOOK)
( InsightsIAS Mock Test Series for UPSC Preliminary Exam 2020 )

23 What is the function of Haemoglobin in the human body?


1. Transport of gases
2. Regulating release of enzymes

Which of the above is/are correct?


A. 1 only
B. 2 only
C. Both 1 and 2
D. None

Your Answer : D
Correct Answer : A

Answer Justification :

Justification: Haemoglobin is involved in the transport of other gases: It carries some of the body's
respiratory carbon dioxide (about 20–25% of the total) as carbaminohemoglobin, in which CO2 is
bound to the globin protein.

The molecule also carries the important regulatory molecule nitric oxide bound to a globin protein
thiol group, releasing it at the same time as oxygen

Hemoglobin is also found outside red blood cells and their progenitor lines.

Hemoglobin and hemoglobin-like molecules are also found in many invertebrates, fungi, and plants.

In these organisms, hemoglobins may carry oxygen, or they may act to transport and regulate other
small molecules and ions such as carbon dioxide, nitric oxide, hydrogen sulfide and sulfide.

Q Source: Revision: UPSC based Q papers

24 The Z-Morh tunnel, recently seen in news, will help to ensure year-long road connectivity between

A. Srinagar and Kargil


B. Jammu and Srinagar
C. Kargil and Ambaleshwar
D. Jammu and Xinjiang

Your Answer : A
Correct Answer : A

Answer Justification :

Justification: Z-Morh Tunnel is a road tunnel project and is the first step toward ensuring all-
weather connectivity between Srinagar and Kargil in Ladakh region of India. The tunnel gets its
name from its Z formation between Sonamarg and Gagangir.

The 6.5 km long road tunnel is being constructed near Gagangir or Gagangair in Ganderbal district
Online Prelims TEST - 23 (TEXTBOOK)
( InsightsIAS Mock Test Series for UPSC Preliminary Exam 2020 )

of the Indian union territory of Jammu and Kashmir. It has a high strategic importance for India.
The tunnel would avoid the regions of snowfall and avalanches. Along with Zoji-la Tunnel (which is
22 km from Z-Morh tunnel towards Leh), the Z-Morh tunnel will ensure year-long road connectivity
between Srinagar and Kargil which currently remains closed for about seven months due to snow.

The tunnel will also benefit social and economic development of region apart from boosting the
tourism in Sonmarg region as the area witnesses heavy snowfall during winters.

Once all tunnels are put in place, the time to drive from Srinagar to Leh will be reduced and will not
require overnight halt at Kargil. The entire stretch will become motorable throughout the year.

Q Source:
https://www.insightsonindia.com/2020/01/18/insights-daily-current-affairs-pib-summary-18-january-
2020/

25 Consider the following statements.


The central government has allowed
1. commercial coal mining by private entities.
2. Hundred per cent foreign direct investment (FDI) under the automatic route in coal mining for open
sale

Select the correct answer using the codes below.


A. 1 only
B. 2 only
C. Both 1 and 2
D. None of the above

Your Answer : C
Correct Answer : C

Answer Justification :

Justification: Statement 1: In 2018, the government allowed commercial mining by private entities
and set a mining target of 1.5 billion tonnes by 2020. Out of this, 1 billion tonnes was set to be from
Coal India, while 500 million tonnes was to be from non-Coal India entities. This target has now
been revised to 1 billion tonnes by 223-24.

Government has also approved an ordinance that opens up the sector to players outside steel and
power as well as removes end-use restrictions. The Union Cabinet has approved the promulgation of
Mineral Laws (Amendment) Ordinance 2020 that will amend the Mines and Minerals (Development
and Regulation) Act 1957 and Coal Mines (Special Provisions) Act 2015.

The move is likely to create an efficient energy market and bring in more competition as well as
reduce coal imports. However, the move might put an end to Coal India Ltd's monopoly in the
sector. The move would also help India gain access to high-end technology for underground mining
used by miners across the globe.

Statement 2: In August 2019, the government announced 100 per cent foreign direct investment
Online Prelims TEST - 23 (TEXTBOOK)
( InsightsIAS Mock Test Series for UPSC Preliminary Exam 2020 )

(FDI) under the automatic route in coal mining for open sale, besides creating associated
infrastructure, such as washeries.

Q Source:
https://www.businesstoday.in/sectors/energy/govt-opens-up-coal-mining-sector/story/393407.html

https://www.insightsonindia.com/2020/01/10/insights-daily-current-affairs-pib-summary-10-january-
2020/

26 Consider the following barchart of precipitation and temperature of a particular climate type.

The chart corresponds best to which of the following climate types?


A. Equatorial Climate
B. Tropical Continental Climate
C. Mediterranean climate
D. Marine West Coast climate

Your Answer : C
Correct Answer : C

Answer Justification :

Justification: Option C: Mediterranean climate is characterized by hot, dry summers and cool, wet
winters and located between about 30° and 45° latitude north and south of the Equator and on the
western sides of the continents.

Option B: The following chart refers to such climate.


Online Prelims TEST - 23 (TEXTBOOK)
( InsightsIAS Mock Test Series for UPSC Preliminary Exam 2020 )

Places with this type of climate are located in the centre of continents approximately between
latitudes 50 and 150 north and south of the Equator. It is also found on the higher land Of the East
African Plateau which straddles the Equator, for e.g. northern Nigeria.

The climate has two distinct seasons.

A very warm. dry season when conditions are similar to those of the hot desert

A hot, wet season when the weather more resembles that of equatorial areas

Option D: This features cool summers (relative to their latitude) and cool winters, with a relatively
narrow annual temperature range. The curve above shows warm summers.

Q Source: Additional Research: Chapter 22: GC Leong Geography

27 World Health Organization (WHO) has released a list of 13 urgent global health challenges. These
include
1. Expensive Medicine Patents
2. Delivering Health in Conflict and Crisis
3. Climate Crisis

Select the correct answer using the codes below.


A. 1 and 3 only
B. 2 and 3 only
C. 1, 2 and 3
D. 2 only

Your Answer : C
Correct Answer : B

Answer Justification :

Justification: Some of these include:


Online Prelims TEST - 23 (TEXTBOOK)
( InsightsIAS Mock Test Series for UPSC Preliminary Exam 2020 )

Climate Crisis: Climate change causes more extreme weather events, exacerbates malnutrition,
and fuels the spread of infectious diseases such as malaria. The same emissions that pollute the air
and cause global warming are responsible for more than one quarter of deaths from heart attack,
stroke, lung cancer, and chronic respiratory disease.

Delivering Health in Conflict and Crisis: Last year, most disease outbreaks that required the
highest level of WHO response occurred in countries with protracted conflict. And the “disturbing”
trend in which healthcare workers and facilities are targeted continued. Conflict is also forcing a
record number of people out of their own homes, leaving them with little or no access to healthcare,
often for years.

Healthcare Equality: “Persistent and growing” socioeconomic gaps that result in major
discrepancies in the quality of people’s health is also an urgent challenge. There is not only an 18-
year difference in life expectancy between rich and poor countries, but also a marked gap within
countries and even within cities.

Expanding Access to Medicines: About one third of the world’s people lack access to medicines,
vaccines, diagnostic tools, and other essential health products. Low access to quality health
products threatens health and lives and contributes to drug resistance. This does not talk about
expensive medicine patents though.

Infectious Diseases: Such as HIV, tuberculosis, viral hepatitis, malaria, neglected tropical
diseases, and sexually transmitted infections will take the lives of an estimated four million people
in 2020, most of them poor. Meanwhile, vaccine-preventable diseases continue to kill, including
measles, which took 140,000 lives in 2019. Polio is also once again a concern, with 156 cases of wild
poliovirus last year, the most since 2014.

Preparing for Epidemics: A pandemic of a new, highly infectious, airborne virus — most likely a
strain of influenza— to which most people lack immunity is inevitable. And vector-borne diseases
like dengue, malaria, Zika, chikungunya, and yellow fever are spreading as mosquito populations
move into new areas, fanned by climate change.

Dangerous Products: Lack of food, unsafe food, and unhealthy diets are to blame for nearly one
third of the global disease burden.

Investing in People Who Defend Our Health: Another challenge is the global shortage of healthcare
workers. The world will need 18 million more healthcare workers by 2030, mostly in low- and
middle-income countries, including nine million nurses and midwives.

Q Source:
https://www.insightsonindia.com/2020/01/17/insights-daily-current-affairs-pib-summary-17-january-
2020/

28 G-8 24/7 Network is

A. a point to point network for urgent assistance in cybercrime matters


B. a mechanism for emergency economic aid and transfers
C. a communication node for immediate contact between the government officials of the G-8
Online Prelims TEST - 23 (TEXTBOOK)
( InsightsIAS Mock Test Series for UPSC Preliminary Exam 2020 )

D. None of the above

Your Answer : A
Correct Answer : A

Answer Justification :

Justification: G-8 24/7 Network allows law enforcement agencies to make urgent preservation
requests of the digital data before it perishes.

It is a new mechanism to expedite contacts between the Participating States or other autonomous
law enforcement jurisdictions of a State.

It is a point to point network for urgent assistance in cybercrime matters.

The Union Home Ministry has issued revised guidelines to streamline the process of seeking legal
assistance from foreign countries in criminal matters.

The G-8 countries’ 24/7 Network could be used to place request for preserving data in real time. For
more, please see the Q source link.

Q Source:
https://www.insightsonindia.com/2020/01/10/insights-daily-current-affairs-pib-summary-10-january-
2020/

29 When the cold Labrador current and warm Gulf stream current meet, it leads to
1. Dense mist and fog
2. An Arid climate Zone

Which of the above is/are correct?


A. 1 only
B. 2 only
C. Both 1 and 2
D. None

Your Answer : A
Correct Answer : C

Answer Justification :

Justification: Statement 1: When cold and warm ocean current meet, the warm ocean current
condenses after getting in contact with cold air and it results into fog.

Statement 2: It also leads to high rate of precipitation in the region.


Online Prelims TEST - 23 (TEXTBOOK)
( InsightsIAS Mock Test Series for UPSC Preliminary Exam 2020 )

The circulation in the area where the Labrador Current and Gulf Stream meet is one of the most
complex and variable regions in the world.

Q Source: Chapter 24: Goh Cheng Leong Geography

30 Why the Southern Hemisphere does not experience cold and harsh climate like the Siberia?
1. Relative absence of land in the Southern Hemisphere in the higher latitudes
2. Strong oceanic influence on the climate reduces the severity of winter

Which of the above is/are correct?


A. 1 only
B. 2 only
C. Both 1 and 2
D. None

Your Answer : C
Correct Answer : C

Answer Justification :

Justification: The Siberian climate is conspicuously absent in the Southern Hemisphere because of
the narrowness of the Southern Hemisphere in the high latitudes.

The strong oceanic influence reduces the severity of the winter and coniferous forests are found
only on the mountainous uplands of Southern Chile, New Zealand, Tasmania and SE Australia.

Q Source: Chapter 23: Goh Cheng Leong Geography

31 With reference to LIGO – India project, consider the following statements.


1. It is a planned advanced gravitational-wave observatory to be located in India as part of the
worldwide LIGO network.
Online Prelims TEST - 23 (TEXTBOOK)
( InsightsIAS Mock Test Series for UPSC Preliminary Exam 2020 )

2. It is piloted by Department of Atomic Energy (DAE) and Department of Science and Technology
(DST).

Select the correct answer using the codes below.


A. 1 only
B. 2 only
C. Both 1 and 2
D. None

Your Answer : A
Correct Answer : C

Answer Justification :

Justification: Statement 1: The Laser Interferometer Gravitational-Wave Observatory (LIGO) -


India is a planned advanced gravitational-wave observatory to be located in India as part of the
worldwide network, whose concept proposal is now under active consideration in India and the
USA. LIGO-India is envisaged as a collaborative project between a consortium of Indian research
institutions and the LIGO Laboratory in USA, along with its international partners.

LIGO-India received the Indian Government's in-principle approval in February 2016. Since then the
project reached several milestones towards selecting and acquiring a site and building the
observatory.

Statement 2: The LIGO-India project will be built by the Department of Atomic Energy (DAE) and
the Department of Science and Technology (DST), Government of India, with a Memorandum of
Understanding (MoU) with the National Science Foundation (NSF), USA, along with several
national and international research and academic institutions.

The LIGO-India project will be jointly coordinated and executed by three Indian research
institutions: the Inter-University Centre for Astronomy and Astrophysics (IUCAA), Pune and
Department of Atomic Energy organisations: Institute for Plasma Research (IPR), Gandhinagar and
the Raja Ramanna Centre for Advanced Technology (RRCAT), Indore.

Q Source:
https://www.insightsonindia.com/2020/01/09/insights-daily-current-affairs-pib-summary-09-january-
2020/

https://www.ligo-india.in/

32 What climatic conditions in the Mediterranean region support vine cultivation?


1. Long growing seasons of moderate to warm temperatures
2. Very low precipitation in grape growing seasons

Which of the above is correct?


A. 1 only
B. 2 only
C. Both 1 and 2
Online Prelims TEST - 23 (TEXTBOOK)
( InsightsIAS Mock Test Series for UPSC Preliminary Exam 2020 )

D. None

Your Answer : C
Correct Answer : A

Answer Justification :

Justification: The presence of adequate sun, heat and water are all vital to the healthy growth and
development of grapevines during the growing season.

In general, grapevines thrive in temperate climates which grant the vines long, warm periods
during the crucial flowering, fruit set and ripening periods.

On average, a grapevine needs around 710 mm (28 in) of water for sustenance during the growing
season, not all of which may be provided by natural rain fall. In Mediterranean and many
continental climates, the climate during the growing season may be quite dry and require additional
irrigation. So, 2 is incorrect.

However, the advantage of Mediterranean agriculture is that more valuable crops such as fruits and
vegetables are grown in winters when there is great demand in European and North American
markets

Q Source: AR: Chapter 22: GC Leong Geography

33 Which of the following are some of most common rare diseases recorded in India?
1. Haemophilia
2. Thalassemia
3. Sickle-cell anaemia

Select the correct answer using the codes below.


A. 1 only
B. 2 and 3 only
C. 3 only
D. 1, 2 and 3

Your Answer : B
Correct Answer : D

Answer Justification :

Justification: A rare disease, also referred to as an orphan disease, is any disease that affects a
small percentage of the population. While there is no universally accepted definition of rare
diseases, countries typically arrive at their own descriptions, taking into consideration disease
prevalence, its severity and the existence of alternative therapeutic options.

Characteristics:

Rare diseases are characterised by a wide diversity of symptoms and signs that vary not only from
Online Prelims TEST - 23 (TEXTBOOK)
( InsightsIAS Mock Test Series for UPSC Preliminary Exam 2020 )

disease to disease but also from patient to patient suffering from the same disease. Relatively
common symptoms can hide underlying rare diseases, leading to misdiagnosis.

The most common rare diseases recorded in India are Haemophilia, Thalassemia, sickle-cell
anaemia and primary immuno deficiency in children, auto-immune diseases, Lysosomal storage
disorders such as Pompe disease, Hirschsprung disease, Gaucher’s disease, Cystic Fibrosis,
Hemangiomas and certain forms of muscular dystrophies.

Q Source:
https://www.insightsonindia.com/2020/01/15/insights-daily-current-affairs-pib-summary-15-january-
2020/

34 Which of the following is one of the seven major classical dance styles of India?

A. Theyyam
B. Sattriya
C. Kalaripayattu
D. Lazim

Your Answer : B
Correct Answer : B

Answer Justification :

Justification: Options A, C and D: There are several forms of dance-drama or folk theatre, such as
the Nautanki of Rajasthan, Uttar Pradesh and Bihar, the Bhavai of Gujarat, the irreverent Tamasha
of Maharashtra , the Bengali Jatra, the spectacular Yakshagana of Karnataka and Theyyam of
Kerala, all of which narrate legends of local heroes, kings and deities. Martial art forms throughout
the country have been stylized to quasi dance forms, notable among which are the martial dances of
the North-eastern hill tribes, the Lazim dances of Maharashtra, the Kalaripayattu of Kerala, and the
highly stylized masked Chhau dances of Orissa, West Bengal and Bihar.

Option B: There are seven major classical dance styles — Bharatnatyam from Tamil Nadu and
Karnataka, Kathakali, a classical dance-drama from Kerala, Manipuri from Manipur, Kathak from
Uttar Pradesh, Odissi from Orissa, and Kuchipudi from Andhra Pradesh and Sattriya from Assam
which has recently been included in the fold of Classical Dances.

Q Source: http://ccrtindia.gov.in/performingart.php

35 State Energy Efficiency Index 2019 has been released. Consider the following about it.
1. The index has developed by the Bureau of Energy Efficiency (BEE) in association with the Alliance
for an Energy Efficient Economy (AEEE).
2. Manipur was the best performer in the latest index.
3. It does not take the performance of Union Territories (UTs) into account.

Select the correct answer using the codes below.


A. 1 and 2 only
Online Prelims TEST - 23 (TEXTBOOK)
( InsightsIAS Mock Test Series for UPSC Preliminary Exam 2020 )

B. 2 and 3 only
C. 1 only
D. 1 and 3 only

Your Answer : C
Correct Answer : C

Answer Justification :

Justification: The first such Index, the “State Energy Efficiency Preparedness Index 2018”, was
launched on August 1, 2018.

About the 2019 index:

The index has developed by the Bureau of Energy Efficiency (BEE) in association with the Alliance
for an Energy Efficient Economy (AEEE).

Performance of various states:

Karnataka, HP, Haryana and Puducherry were the best performers.

Manipur, Jharkhand, Rajasthan and Jammu and Kashmir were the least performers.

It tracks the progress of Energy Efficiency (EE) initiatives in 36 states and union territories based
on 97 significant indicators.

Learning: For a rational comparison, the States or Union Territories are grouped into four groups
based on the aggregated Total Primary Energy Supply (TPES) required to meet the state’s actual
energy demand (electricity, coal, oil, gas, among others) across sectors.

The Index categorises states as ‘Front Runner’, ‘Achiever’, ‘Contender’ and ‘Aspirant’ based on
their efforts and achievements towards energy efficiency implementation.

There isn’t any ‘front runner’ state this year.

The objective behind releasing this Index is to help states in contributing towards national goals on
energy security and climate action by:

Helping drive energy efficiency policies and program implementation at the state and local
level.

Tracking progress in managing the states’ and India’s energy footprint.

Institutionalising data capture and monitoring of energy efficiency activities by states.


Online Prelims TEST - 23 (TEXTBOOK)
( InsightsIAS Mock Test Series for UPSC Preliminary Exam 2020 )

Q Source:
https://www.insightsonindia.com/2020/01/11/insights-daily-current-affairs-pib-summary-11-january-
2020/

36 Consider the following statements.


1. No river drains in the Arctic Ocean.
2. Arctic Ocean is at a higher average elevation than the Northern Hinterland of Eurasia.

Which of above is/are correct?


A. 1 only
B. 2 only
C. Both 1 and 2
D. None of the above

Your Answer : D
Correct Answer : D

Answer Justification :

Justification: Unfortunately, over the greater part of Siberia, all the rivers drain polewards into the
Arctic ocean which is frozen for the three-quarters of the years.

With the use of Northern Sea route, which links Murmansk and Vladivostok via the Arctic Ocean,
development is increasing.

Cheap hydro-electricity for driving the saw-mills is harnessed in the mountainous uplands of North
America and Europe and has greatly assisted the lumbering industry.

Q Source: Chapter 23: Goh Cheng Leong Geography

37 Which of these are species of hardwood trees?


1. Birch
2. Mahogany
3. Spruce
4. Redwood

Select the correct answer using the codes below.


A. 1 and 2 only
B. 2, 3 and 4 only
C. 1 and 3 only
D. 1, 2, 3 and 4

Your Answer : D
Correct Answer : A

Answer Justification :
Online Prelims TEST - 23 (TEXTBOOK)
( InsightsIAS Mock Test Series for UPSC Preliminary Exam 2020 )

Justification: Hardwood is wood from dicot trees. These are usually found in broad-leaved
temperate and tropical forests. In temperate and boreal latitudes they are mostly deciduous, but in
tropics and subtropics mostly evergreen.

Hardwoods are produced by angiosperm trees that reproduce by flowers, and have broad leaves.

Statement 2: Mahogany is a tropical hardwood species indigenous to the Americas.

Ash, Alder, Aspen, Balsa, Beech, Birch, Cherry and Chestnut are some other hardwood trees.

Q Source: Chapter 22: Goh Cheng Leong Geography

38 What is common between Chakiarkoothu, Koodiyattam, Krishnattam and Ramanattam?


1. They are all ritual performing arts.
2. They originate from Kerala.
3. They have had some influence on Kathakali in its form and technique.

Select the correct answer using the codes below.


A. 2 only
B. 1 and 2 only
C. 1 and 3 only
D. 1, 2 and 3

Your Answer : C
Correct Answer : D

Answer Justification :

Justification: Kathakali, as a dance form popular today, is considered to be of comparatively recent


origin. However, it is an art which has evolved from many social and religious theatrical forms
which existed in the southern region in ancient times.

Chakiarkoothu, Koodiyattam, Krishnattam and Ramanattam are few of the ritual performing arts of
Kerala which have had a direct influence on Kathakali in its form and technique. Legend has it that
the refusal of the Zamorin of Calicut to send his Krishnattam troupe to Travancore, so enraged the
Raja of Kottarakkara, that he was inspired to compose the Ramanattam.

Q Source: http://ccrtindia.gov.in/kathakali.php

39 With reference to the Societies Registration Act, 1860, consider the following statements.
1. Societies can be formed only for literary or charitable purposes.
2. The Act and related bodies are exempt from the Right to Information (RTI) Act.
3. Bodies under the act can be formed by way of a memorandum of association (MoA) and do not
necessarily require formal legislation.

Select the correct answer using the codes below.


A. 1 and 2 only
Online Prelims TEST - 23 (TEXTBOOK)
( InsightsIAS Mock Test Series for UPSC Preliminary Exam 2020 )

B. 2 only
C. 3 only
D. 1 and 3 only

Your Answer : C
Correct Answer : C

Answer Justification :

Justification: The Act continues until today and being an Act of Parliament, comes under the RTI
Act, wherein the government is legally responsible to give any information requested by any citizen
of India with respect to any society

Under the Act societies may be formed, by way of a memorandum of association, by any seven or
more people associated for any literary, scientific or charitable purpose.

The act thus allows the registration of entities generally involved in the benefit of society -
education, health, employment etc.

Q Source: Revision: IYB: http://www.mca.gov.in/MinistryV2/societiesregistrationact.html

40 Fundamental rights that have a bearing on Education include


1. Article 13: Powers of judicial review
2. Article 15: Prohibition of discrimination on certain grounds
3. Article 21A: Inserted by 86th Constitutional amendment act
4. Article 28: Freedom of Religion
5. Article 30: Certain educational rights of minorities

Select the correct answer using the codes below.


A. 2, 3 and 4 only
B. 1, 2, 3, 4 and 5
C. 3, 4 and 5 only
D. 1, 2 and 5 only

Your Answer : B
Correct Answer : B

Answer Justification :

Justification: All these are Fundamental Rights under Part III.

A13. Laws inconsistent with or in derogation of the fundamental rights, where the court can
cut down a law that infringes on the right to education.

A15. Prohibition of discrimination on grounds of religion, race, caste, sex or place of birth that
is important for inclusive education
Online Prelims TEST - 23 (TEXTBOOK)
( InsightsIAS Mock Test Series for UPSC Preliminary Exam 2020 )

A21A. Right to education [Inserted by the 86th Amendment in December, 2002 and passed by
the Parliament in July, 2009. The provisions of the Act came into force from 1st April, 2010]

A28. Freedom as to attendance at religious instruction or religious worship in certain


educational institutions

A30. Right of minorities to establish and administer educational institutions

Q Source: http://mhrd.gov.in/constitutional-provision

41 Consider the following statements about the Intergovernmental Panel on Climate Change (IPCC).
1. It is a scientific and intergovernmental body under the auspices of the United Nations.
2. Membership of the IPCC is open to all members of the World Meteorological Organization (WMO)
and the United Nations Environment Program (UNEP).

Which of the above is/are correct?


A. 1 only
B. 2 only
C. Both 1 and 2
D. None of the above

Your Answer : D
Correct Answer : C

Answer Justification :

Learning: It was set up at the request of member governments, dedicated to the task of providing
the world with an objective, scientific view of climate change and its political and economic impacts.

It was first established in 1988 by two United Nations organizations, the World Meteorological
Organization (WMO) and the United Nations Environment Programme (UNEP), and later endorsed
by the United Nations General Assembly.
Online Prelims TEST - 23 (TEXTBOOK)
( InsightsIAS Mock Test Series for UPSC Preliminary Exam 2020 )

Membership of the IPCC is open to all members of the WMO and UNEP.

The IPCC produces reports that support the United Nations Framework Convention on Climate
Change (UNFCCC), which is the main international treaty on climate change

Q Source: Major bodies: Climate Change and mitigation

42 Winters are abnormally mild in North-West European Maritime Climate due to


1. Warm currents of North Atlantic Drift
2. Frontal cyclonic activity

Select the correct answer using the codes below.


A. 1 only
B. 2 only
C. Both 1 and 2
D. None of the above

Your Answer : C
Correct Answer : A

Answer Justification :

Justification: It is also called as the British Climate type. The cool temperate western margins
are under the influence of the Westerlies all-round the year.

They are the regions of frontal cyclonic activity, i.e. Temperate Cyclones, but these are not the
reason for an abnormally mild winter (reason below).

This type of climate is typical to Britain, hence the name ‘British Type’.

It is called as North-West European Maritime Climate due to greater oceanic influence.

The mean annual temperatures are usually between 5° C and 15° C.

Winters are abnormally mild. This is because of the warming effect brought by warm North
Atlantic Drift.

Sometimes, unusual cold spells are caused by the invasion of cold polar continental air (Polar
Vortex) from the interiors.

Q Source: Chapter 22-23: Goh Cheng Leong: Certificate Physical and Human Geography

43 Nations sharing a coastline with the Red Sea include


Online Prelims TEST - 23 (TEXTBOOK)
( InsightsIAS Mock Test Series for UPSC Preliminary Exam 2020 )

1. Jordan
2. Saudi Arabia
3. Egypt
4. Oman

Which of the above is/are correct?


A. 1 and 2 only
B. 2 and 3 only
C. 2, 3 and 4 only
D. 1 and 4 only

Your Answer : C
Correct Answer : B

Answer Justification :

Learning: Oman is towards the side of Persian Gulf and Jordan is landlocked.

Q Source: Map-based questions: West Asia

44 Forest Advisory Committee has recently approved the implementation of Green Credit scheme.
Consider the following about it.
1. It allows forests to be traded as a commodity.
2. It allows the Forest Department to outsource its responsibilities of reforesting to non-government
Online Prelims TEST - 23 (TEXTBOOK)
( InsightsIAS Mock Test Series for UPSC Preliminary Exam 2020 )

agencies.

Select the correct answer using the codes below.


A. 1 only
B. 2 only
C. Both 1 and 2
D. None of the above

Your Answer : C
Correct Answer : C

Answer Justification :

Justification: The scheme allows agencies — they could be private companies, village forest
communities — to identify land and begin growing plantations.

After three years, they would be eligible to be considered as compensatory forest land if they met
the Forest Department’s criteria.

An industry needing forest land could then approach the agency and pay it for parcels of such
forested land, and this would then be transferred to the Forest Department and be recorded as
forest land.

The participating agency will be free to trade its asset, that is plantation, in parcels, with project
proponents who need forest land.

Present scenario:

In the current system, industry needs to make good the loss of forest by finding appropriate
non-forest land — equal to that which would be razed.

It also must pay the State Forest Department the current economic equivalent — called Net
Present Value — of the forest land.

It’s then the Forest Department’s responsibility to grow appropriate vegetation that, over
time, would grow into forests.

Q Source:
https://www.insightsonindia.com/2020/01/10/insights-daily-current-affairs-pib-summary-10-january-
2020/

45 It is the only protected area that is a preferred habitat for Hoolock Gibbons, the only ape found in
India:

A. Nokrek National Park


Online Prelims TEST - 23 (TEXTBOOK)
( InsightsIAS Mock Test Series for UPSC Preliminary Exam 2020 )

B. Simlipal Wildlife Sanctuary


C. Kaziranga National Park
D. Namdapha National Park

Your Answer : D
Correct Answer : D

Answer Justification :

Learning: It is the largest protected area in the Eastern Himalaya biodiversity hotspot and is
located in Arunachal Pradesh.

It is also the third largest national park in India in terms of area. The park harbours the
northernmost lowland evergreen rainforests in the world at sub-tropical latitude.

The habitat changes with increasing altitude from tropical moist forests to Montane forests,
temperate forests and at the higher elevations, to Alpine meadows and perennial snow.

The park has extensive bamboo forests and secondary forests in addition to the primary forests.

Q Source: Test Syllabus: National Parks, Biosphere Reserves

46 Sea Guardians 2020, recently seen in news, is a joint naval drill between

A. India and China


B. USA and Singapore
C. Pakistan and China
D. Japan and South Korea

Your Answer : B
Correct Answer : C

Answer Justification :

Justification: It is a joint naval drill between Pakistan and China. The latest edition is being held in
Karachi.

It is the sixth in the bilateral series. It will focus on “augmenting interoperability and strategic
cooperation.”

It is important for India given the geo-political risks carried by Pak-China cooperation in the military
arena.

Q Source:
https://www.insightsonindia.com/2020/01/10/insights-daily-current-affairs-pib-summary-10-january-
2020/
Online Prelims TEST - 23 (TEXTBOOK)
( InsightsIAS Mock Test Series for UPSC Preliminary Exam 2020 )

47 Consider the following statements.


Despite being in the cool temperate region, Western Europe does not experience extreme climate
because
1. The western margins of Europe are under the influence of easterlies throughout the year.
2. The North Atlantic drift moderates the overall temperature.

Which of the above is/are correct?


A. 1 only
B. 2 only
C. Both 1 and 2
D. None of the above

Your Answer : C
Correct Answer : B

Answer Justification :

Justification: The climate of Western Europe is a marine climate. Its abundant moisture comes
from the Atlantic Ocean. The ocean moderates the temperature during the seasons, making the
winter moist and mild, and the summer moist and cool. This climate primarily stays to the north of
the Alps, thus considering the Alps a climatic divide.

Statement 1: It is under the influence of westerlies throughout the year.

Statement 2: The relatively warm waters of the North Atlantic Drift are responsible for moderating
the climate of western Europe, so that winters are less cold than would otherwise be expected at its
latitude

Q Source: Chapter 22: Goh Cheng Leong Geography

48 With reference to Operation Sankalp, seen in news recently, consider the following statements.
1. It is a maritime security operation launched by India to promptly respond to the emergency
situations for Indian sea vessels in the Middle East region.
2. It has been launched in order to ensure safe passage of Indian Flag Vessels through the Strait of
Hormuz through the deployment of Indian navy warships and aircraft.

Select the correct answer using the codes below.


A. 1 only
B. 2 only
C. Both 1 and 2
D. None of the above

Your Answer : D
Correct Answer : C

Answer Justification :

Justification: In the backdrop of the deteriorating security situation in the Gulf region, post attacks
Online Prelims TEST - 23 (TEXTBOOK)
( InsightsIAS Mock Test Series for UPSC Preliminary Exam 2020 )

on merchant ships in the Gulf of Oman in Jun 19, Indian Navy had commenced Maritime Security
Operations, code named Op SANKALP, in the Gulf Region in 2019 to ensure safe passage of Indian
Flag Vessels transiting through the Strait of Hormuz.

Indian Navy warships and aircraft were deployed to establish presence, provide a sense of
reassurance to the Indian merchantmen, monitor the ongoing situation and respond to any
emergent crises. One war ship is presently deployed for Operation SANKALP.

The operation is being progressed in close coordination with all stakeholders including Ministry of
Defence, Ministry of External Affairs, Ministry of Shipping, Ministry of Petroleum and Natural Gas
and DG, Shipping. Regular Inter-Ministerial meetings have been held to review the evolving
Maritime Security Situation in the Gulf region.

Indian Navy continues to monitor the situation in the Gulf region and is maintaining presence in the
region to ensure security of our sea borne trade and the safety of Indian Flag Merchant Vessels
transiting through the region.

Q Source: https://pib.gov.in/newsite/PrintRelease.aspx?relid=197334

https://www.insightsonindia.com/2020/01/10/insights-daily-current-affairs-pib-summary-10-january-
2020/

49 Consider the following about Seaweed.


1. Size of seaweeds range from that of a phytoplankton to a kelp forest.
2. They are rich in vitamins, minerals and fibre.
3. Multi-cellular algae cannot be characterized as seaweed.

Select the correct answer using the codes below.


A. 1 only
B. 2 and 3 only
C. 1 and 2 only
D. 1 and 3 only

Your Answer : C
Correct Answer : C

Answer Justification :

Justification: A seaweed may belong to one of several groups of multi-cellular algae: the red algae,
green algae, and brown algae.

Many seaweeds contain anti-inflammatory and anti-microbial agents. Their known medicinal
effects have been legion for thousands of years; the ancient Romans used them to treat
wounds, burns, and rashes.

Anecdotal evidence also suggests that the ancient Egyptians may have used them as a
treatment for breast cancer.
Online Prelims TEST - 23 (TEXTBOOK)
( InsightsIAS Mock Test Series for UPSC Preliminary Exam 2020 )

Some seaweeds are microscopic, such as the phytoplankton that live suspended in the water
column and provide the base for most marine food chains.

Some are enormous, like the giant kelp that grow in abundant “forests” and tower like
underwater redwoods from their roots at the bottom of the sea.

Most are medium-sized, come in colors of red, green, brown, and black, and randomly wash
up on beaches and shorelines just about everywhere.

Q Source: Chapter 24: Goh Cheng Leong Geography

50 Consider the following statements.


1. Low pressure systems are usually characterized by dry and settled weather.
2. High pressure systems usually bring wind and precipitation.

Which of the above is/are correct?


A. 1 only
B. 2 only
C. Both 1 and 2
D. None

Your Answer : D
Correct Answer : D

Answer Justification :

Justification: Statement 1: Low pressure systems attract wind from high pressure areas nearby,
sometimes even forming cyclonic depressions.

Pressure belts are created by rising warm air and descending cool air.

Q Source: Additional Research: Chapter 22: GC Leong Geography

51 Which of the following climate types is NOT witnessed in Europe?


Online Prelims TEST - 23 (TEXTBOOK)
( InsightsIAS Mock Test Series for UPSC Preliminary Exam 2020 )

A. Temperate maritime
B. Continental Interior
C. Mediterranean Climate
D. Humid Subtropical climate

Your Answer : B
Correct Answer : D

Answer Justification :

Justification: Option A and option C: These climate types have been explained in another question.

Option B: The continental interior stretches from central Europe westwards into Russia. The climate
here is one of hot damp summers and very cold dry winters.

The main reason for this climate's characteristics is distance from the Sea. Inland areas are warmer
in summer and colder in Winter than coastal places. Being far from the sea they also tend to be
drier.

Q Source: Chapter 23: GC Leong Geography

52 As per classical texts, to present nritya and natya effectively, a dancer should be trained to
communicate the navarasas. Which of the following is NOT a navarasa?
Online Prelims TEST - 23 (TEXTBOOK)
( InsightsIAS Mock Test Series for UPSC Preliminary Exam 2020 )

A. Karuna
B. Bibhatsa
C. Shaanta
D. Lasya

Your Answer : D
Correct Answer : D

Answer Justification :

Justification: As per the ancient treatises, dance is considered as having three aspects: natya,
nritya and nritta. Natya highlights the dramatic element and most dance forms do not give
emphasis to this aspect today with the exception of dance-drama forms like Kathakali. Nrityais
essentially expressional, performed specifically to convey the meaning of a theme or idea. Nritta on
the other hand, is pure dance where body movements do not express any mood (bhava), nor do they
convey any meaning. To present nritya and natya effectively, a dancer should be trained to
communicate the navarasas. These are: love (shringaara), mirth (haasya), compassion (karuna),
valour(veera), anger (roudra), fear (bhayanak), disgust (bibhatsa), wonder (adbhuta) and peace
(shaanta).

Option D: An ancient classification followed in all styles is of Tandava and Lasya. Tandava the
masculine, is heroic bold and vigorous. Lasya the feminine is soft, lyrical and graceful. Abhinaya,
broadly means expression. This is achieved through angika, the body and limbs, vachikasong and
speech and aharya, costume and adornment; and satvika, moods and emotions.

Q Source: http://ccrtindia.gov.in/classicaldances.php

53 Which of the following are landlocked states in Europe?

A. Poland, Switzerland, Austria, Hungary, Luxembourg


B. Czech Republic, Belgium, Austria, Hungary, Luxembourg
C. Switzerland, Austria, Hungary, Czech Republic, Luxembourg
D. Czech Republic, Switzerland, Austria, Hungary, Denmark

Your Answer : C
Correct Answer : C

Answer Justification :

Learning: There are 17 landlocked countries in Europe: Andorra, Armenia, Austria, Belarus, Bosnia
and Herzegovina, Czech Republic, Hungary, Kazakhstan, Liechtenstein, Luxembourg, Macedonia,
Moldova, San Marino, Serbia, Slovakia, Switzerland and Vatican City.
Online Prelims TEST - 23 (TEXTBOOK)
( InsightsIAS Mock Test Series for UPSC Preliminary Exam 2020 )

Q Source: Map based questions: Europe: Based on Chapter 22-23: GC Leong Geography

54 The language that was declared ‘Classical’ most recently among the following is

A. Malyalam
B. Kannada
C. Odia
D. Sanskrit

Your Answer : C
Correct Answer : C

Answer Justification :

Justification: Currently, six languages enjoy the ‘Classical’ status: Tamil (declared in 2004),
Sanskrit (2005), Kannada (2008), Telugu (2008), Malayalam (2013), and Odia (2014).

Guidelines for declaring a language as ‘Classical’ are:

High antiquity of its early texts/recorded history over a period of 1500-2000 years.

A body of ancient literature/texts, which is considered a valuable heritage by generations of


speakers.

The literary tradition be original and not borrowed from another speech community.

The classical language and literature being distinct from modern, there may also be a
discontinuity between the classical language and its later forms or its offshoots.

How are the Classical languages promoted? Various benefits:

Two major annual international awards for scholars of eminence in classical Indian languages.

A Centre of Excellence for studies in Classical Languages is set up.


Online Prelims TEST - 23 (TEXTBOOK)
( InsightsIAS Mock Test Series for UPSC Preliminary Exam 2020 )

The University Grants Commission is requested to create, to start with at least in the Central
Universities, a certain number of Professional Chairs for the Classical Languages so declared.

At the recently concluded 93rd edition of the Akhil Bharatiya Marathi Sahitya Sammelan, a
resolution was passed demanding the declaration of Marathi as a ‘Classical’ language.

Q Source:
https://www.insightsonindia.com/2020/01/15/insights-daily-current-affairs-pib-summary-15-january-
2020/

55 In which of the following regions extensive commercial grain cultivation is practised?


1. Interior parts of Southeast Asia
2. Pampas of Argentina
3. European Steppes
4. Amazon Basin

Select the correct answer using the codes below.


A. 1 and 2 only
B. 2 and 3 only
C. 1 and 3 only
D. 2, 3 and 4 only

Your Answer : C
Correct Answer : B

Answer Justification :

Justification: Statement 1 and 4: Gathering is practised in: (i) high latitude zones which include
northern Canada, northern Eurasia and southern Chile; (ii) Low latitude zones such as the Amazon
Basin, tropical Africa, Northern fringe of Australia and the interior parts of Southeast Asia.

Statement 2 and 3: Extensive commercial grain cultivation is best developed in Eurasian steppes,
the Canadian and American Prairies, the Pampas of Argentina, the Velds of South Africa, the
Australian Downs and the Canterbury Plains of New Zealand.
Online Prelims TEST - 23 (TEXTBOOK)
( InsightsIAS Mock Test Series for UPSC Preliminary Exam 2020 )

Q Source: Additional Research: Chapter 23: GC Leong Geography

56 Which of the following texts talk about use of masks in theatre performances?

A. Silappadikaram
B. Natyashashtra
C. Manimekalai
D. Akshara-Laksha

Your Answer : D
Correct Answer : B

Answer Justification :

Justification: Option B: The Natya Shastra speaks of masks and their use in theatre. Here it is
mentioned that masks can be made of ground paddy husks applied to cloth.

The earliest traces of human existence go back to the period between 3,00,000 and 2,00,000 BC.

The modern human being first appeared around 36000 BC. The origin of the mask has been traced
to pre-historic man.

Depictions of masks have been found in various rock paintings and cave paintings. Masks were
probably used for hunting and taming animals and for their primeval dances.

Excavations have revealed small hollow masks dating back to the Indus Valley Civilisation. In fact in
Bihar a terracotta mask of the fourth century has also been excavated.

Q Source: CCRT: Performing arts


Online Prelims TEST - 23 (TEXTBOOK)
( InsightsIAS Mock Test Series for UPSC Preliminary Exam 2020 )

57 Regions with cool temperate British type climate can be found in


1. Eastern United States
2. Southern Chile
3. South Africa
4. East Asia

Select the correct answer using the codes below.


A. 1 and 2 only
B. 2 only
C. 3 and 4 only
D. 2 and 4 only

Your Answer : C
Correct Answer : B

Answer Justification :

Justification: The climate type has been described in another question.

Q Source: Chapter 22: Goh Cheng Leong Geography

58 Consider the following statements.


1. Chhau dance is a genre of Indian tribal martial dance.
2. Chhau is performed exclusively by men from the triangular region where Bihar, Bengal and Orissa
meet.
3. Ministry of Culture, Government of India, recognized Chhau as a ‘classical’ dance form.

Which of the above is/are correct?


A. 1 and 2 only
B. 2 and 3 only
C. 1, 2 and 3
D. 1 only

Your Answer : D
Online Prelims TEST - 23 (TEXTBOOK)
( InsightsIAS Mock Test Series for UPSC Preliminary Exam 2020 )

Correct Answer : C

Answer Justification :

Learning: Chhau is a style performed exclusively by men from the triangular area where Bihar,
Bengal and Orissa meet. This is the tribal belt of India home to the tribal groups of Bhulya,
Santhals, Mundas, Hos and Oraons.

The number of recognized classical dances range from eight to more, depending on the source and
scholar. The Sangeet Natak Akademi recognizes eight – Bharatanatyam, Kathak, Kuchipudi, Odissi,
Kathakali, Sattriya, Manipuri and Mohiniyattam. Scholars such as Drid Williams add Chhau,
Yakshagana and Bhagavata Mela to the list. The Culture Ministry of the Government of India
includes Chhau in its classical list.

In the olden days the powerful Ganga and Gajapati rulers of Orissa extended their territory from the
river Ganges in the north to the Godavari in the south with the help of a vast army of valiant Paikas.

They were not in the regular payroll of the army, but received huge land grants from the kings and
the chieftains. They formed the rank of a peasant-militia. Though agriculture was their main
occupation they used to keep themselves prepared by regular practice and training in war
techniques.

Q Source: CCRT: Performing arts

59 Consider the following about the Biodiversity Indicators Partnership (BIP).


1. BIP coordinates the development and delivery of biodiversity indicators for use by the Convention
on Biological Diversity (CBD).
2. Its secretariat is hosted under the International Union for Conservation of Nature (IUCN).

Select the correct answer using the codes below.


A. 1 only
B. 2 only
C. Both 1 and 2
D. None of the above

Your Answer :
Correct Answer : A

Answer Justification :

Justification: The primary role of BIP is to serve the global user community by responding to the
indicator requests of the CBD and other biodiversity-related Conventions, for IPBES, for reporting
on the Sustainable Development Goals, and for use by national and regional governments.

The BIP aims to:

Support the development and use of indicators to measure progress in achieving all the Aichi
Biodiversity Targets of the Strategic Plan for Biodiversity 2011-2020.
Online Prelims TEST - 23 (TEXTBOOK)
( InsightsIAS Mock Test Series for UPSC Preliminary Exam 2020 )

Support the development and use of biodiversity-related indicators in progress reporting of


other Multilateral Environmental Agreements (MEAs), as well as to support statistical
requirements, intergovernmental processes and development processes, including the
Intergovernmental Platform for Biodiversity and Ecosystem Services (IPBES) and the
Sustainable Development Goals (SDGs).

Strengthen capacity at the national level for indicator development and use in implementation
and reporting of National Biodiversity Strategies and Action Plans (NBSAPs) and the SDGs.

The Partnership currently brings together over fifty organizations working internationally on
indicator development to provide the most comprehensive information on biodiversity trends.

As well as the Partners, the Partnership has a secretariat, hosted at UNEP-WCMC, and a Steering
Committee.

Q Source: AR: Revision: IYB: Chapter on Environment

https://www.bipindicators.net/about

60 Consider the following statements about Bharatnatyam.


1. It is known to be ekaharya, where one dancer takes on many roles in a single performance.
2. Abhinaya Darpana, a text by Bharat Muni, mentions Bharatnatyam.
3. Some of the performers and gurus of the dance tradition belong to the devadasi families.

Select the correct answer using the codes below.


A. 2 and 3 only
B. 1, 2 and 3
C. 1 and 3 only
D. 1 only

Your Answer : C
Correct Answer : C

Answer Justification :

Justification: Statement 1: Bharatnatyam dance is known to be ekaharya, where one dancer takes
on many roles in a single performance. In the early 19th century, the famous Tanjore Quartette,
under the patronage of Raja Serfoji are said to have been responsible for the repertoire of
Bharatnatyam dance as we see it today.

Statement 2: Bharatnatyam Dance is considered to be over 2000 years old. Several texts beginning
with Bharata Muni's Natya Shastra (200 B.C.E. to 200 C.E.) provide information on this dance form.
The Abhinaya Darpana by Nandikesvara is one of the main sources of textual material, for the study
of the technique and grammar of body movement in Bharatnatyam Dance. There is also a great deal
of visual evidence of this dance form in paintings and stone and metal sculptures of ancient times.

Statement 3: The style was kept alive by the devadasis, who were young girls 'gifted' by their
Online Prelims TEST - 23 (TEXTBOOK)
( InsightsIAS Mock Test Series for UPSC Preliminary Exam 2020 )

parents to the temples and who were married to the gods. The devadasis performed music and
dance as offerings to the deities, in the temple courtyards. Some of the renowned performers and
gurus of the early part of the century belong to the devadasi families, a well-known name is Bala
Saraswati.

Q Source: http://ccrtindia.gov.in/bharatnatyam.php

61 Cell for IPR Promotion and Management (CIPAM) is a professional body under

A. Ministry of Corporate Affairs


B. Department of Industrial Policy & Promotion (DIPP)
C. NITI Aayog
D. Department of Economic Affairs

Your Answer : C
Correct Answer : B

Answer Justification :

Learning: CIPAM is a professional body under the aegis of the DIPP, Ministry of Commerce &
Industry.

Its mandate is to effectively implement National Intellectual Property Rights (IPR) Policy adopted in
May 2016 with slogan – “Creative India; Innovative India”. It has undertaken several measures to
strengthen IP ecosystem in the country.

It has been mandated to undertake:

Simplifying and streamlining of IP processes by formulating and implementing a focused


strategy for each policy objective

Coordination with State level agencies and Ministries/ Departments of the Government of
India, industry bodies as well as international agencies; IP cells to be set up

IPR awareness campaign across the country in schools, colleges/universities and the industry

Training and sensitization porgrammes for enforcement agencies and Judiciary; coordination
for effective enforcement of IPR rights.

Study and facilitate implementation of best practices for promotion and commercialization of
IP within the country.

Q Source: Revision: IYB: http://cipam.gov.in/cipam/


Online Prelims TEST - 23 (TEXTBOOK)
( InsightsIAS Mock Test Series for UPSC Preliminary Exam 2020 )

62 The Brussels Conference held in the interwar period was found

A. Against Imperialism and for National Independence


B. To settle economic claims of war damages and war crimes in British colonies
C. To establish the British Commonwealth
D. Against capitalism and mass production that exploited labour

Your Answer : C
Correct Answer : A

Answer Justification :

Learning: The Brussels Conference decided to found the League Against Imperialism and for
National Independence. Nehru was elected to the Executive Council of the League.

The Congress also affiliated to the League as an associated member.

At its Calcutta session, the Congress declared that the Indian struggle was a part of the worldwide
struggle against imperialism.

It also decided to open a Foreign Department to develop contacts with other peoples and
movements fighting against imperialism.

It was significant because it brought together representatives and organizations from the
communist world and anti-colonial organizations and activists from the colonized world.

Q Source: Revision: Chapter 30: India’s Struggle for Independence: Bipin Chandra

63 Which of the following best describes ‘Gatka’?

A. A martial art associated with the Sikh history


B. A traditional theatre form of Kashmir
C. Folk theatre of Assam performed by Mizo tribes
D. Narration of mythical plays with emphasis on hand gestures and eye movements

Your Answer : A
Correct Answer : A

Answer Justification :
th
Learning: It an integral part of an array of Sikh Shastar Vidiya developed during 15 century.

It is generally at public display during religious processions.

Gatka is a style of stick fighting between two or more practitioners, with wooden sticks (called Soti)
intended to simulate swords.

It is a unique art to defend & display fighting skills and exercise self-control.
Online Prelims TEST - 23 (TEXTBOOK)
( InsightsIAS Mock Test Series for UPSC Preliminary Exam 2020 )

The Government of Punjab and School Games Federation of India (SGFI) have also incorporated the
Gatka game into the school games.

Q Source: CCRT: Performing arts

64 Consider the following statements.


The subjects covered by Natya Shastra include
1. Dramatic composition
2. Musical composition
3. ‘Rasa’ theory

Select the correct answer using the codes below.


A. 2 only
B. 1 and 3 only
C. 1, 2 and 3
D. 1 and 2 only

Your Answer : C
Correct Answer : C

Answer Justification :

Justification: The text consists of 36 chapters with a cumulative total of 6000 poetic verses
describing performance arts. The subjects covered by the treatise include dramatic composition,
structure of a play and the construction of a stage to host it, genres of acting, body movements,
make up and costumes, role and goals of an art director, the musical scales, musical instruments
and the integration of music with art performance.

The Nāṭya Śāstra is notable as an ancient encyclopaedic treatise on the arts, one which has
influenced dance, music and literary traditions in India. It is also notable for its aesthetic "Rasa"
theory, which asserts that entertainment is a desired effect of performance arts but not the primary
goal, and that the primary goal is to transport the individual in the audience into another parallel
reality, full of wonder, where he experiences the essence of his own consciousness, and reflects on
spiritual and moral questions.

Q Source: AR: http://ccrtindia.gov.in/performingart.php

65 Consider the following statements.


1. The Supreme Court has the power or authority to review any judgment declared by it.
2. A curative petition is the last judicial resort available for redressal of grievances in a court in India.
3. The constitution mentions and provides for curative petition.

Select the correct answer using the codes below.


A. 1 and 2 only
B. 1, 2 and 3
C. 2 and 3 only
D. 1 and 3 only
Online Prelims TEST - 23 (TEXTBOOK)
( InsightsIAS Mock Test Series for UPSC Preliminary Exam 2020 )

Your Answer : A
Correct Answer : A

Answer Justification :

Justification: Article- 137 of the Constitution subjects to the provisions of the guidelines made
under Article 145, by which it is clear that the Supreme Court has the ability to review any
judgment declared by it.

Curative petition is the last judicial resort available for redressal of grievances in court which is
normally decided by judges in-chamber. It is only in rare cases that such petitions are given an
open-court hearing.

S3: But, the concept of curative petition was first evolved by the Supreme Court of India in the
matter of Rupa Ashok Hurra vs. Ashok Hurra and Anr. (2002)where the question was whether an
aggrieved person is entitled to any relief against the final judgement/order of the Supreme Court,
after dismissal of a review petition. It was not mentioned in the constitution.

The Supreme Court in the said case held that in order to prevent abuse of its process and to cure
gross miscarriage of justice, it may reconsider its judgements in exercise of its inherent powers. For
this purpose, the Court has devised what has been termed as a “curative” petition.

To entertain the curative petitions, the Supreme Court has laid down certain specific conditions:

The petitioner will have to establish that there was a genuine violation of principles of natural
justice and fear of the bias of the judge and judgement that adversely affected him.

The petition shall state specifically that the grounds mentioned had been taken in the review
petition and that it was dismissed by circulation.

The petition is to be sent to the three senior most judges and judges of the bench who passed
the judgement affecting the petition, if available.

If the majority of the judges on the above bench agree that the matter needs hearing, then it
would be sent to the same bench (as far as possible) and the court could impose “exemplary
costs” to the petitioner if his plea lacks merit.

Curative petitions have been filed in the Supreme Court by two convicts in the Nirbhaya case. The
petitions come just days after a Delhi sessions court scheduled the execution of the four convicts at
Tihar Jail on January 22.

Q Source: Frequently in news

66 With reference to reproduction in plants, Heterogamy is the practice of


Online Prelims TEST - 23 (TEXTBOOK)
( InsightsIAS Mock Test Series for UPSC Preliminary Exam 2020 )

A. Alternating between the sexual and asexual strategies


B. Reproduction that does not involve a male gamete
C. Formation of a new sporophyte without fertilization
D. Giving rise to reproductive cells, mitospores, that develop into a new organism after dispersal

Your Answer : C
Correct Answer : A

Answer Justification :

Learning: Alternation is observed in several species and a few types of insects, such as aphids
which will, under certain conditions, produce eggs that have not gone through meiosis, thus cloning
themselves.

The cape bee Apis mellifera subsp. capensis can reproduce asexually through a process called
thelytoky. A few species of amphibians, reptiles, and birds have a similar ability (see
parthenogenesis for examples).

For example, the freshwater crustacean Daphnia reproduces by parthenogenesis in the spring to
rapidly populate ponds, then switches to sexual reproduction as the intensity of competition and
predation increases.

Q Source: Revision: UPSC based Q papers

67 Which of the following dance forms has/have been influenced by the Vaishnavite cult?
1. Kathak classical dance
2. Manipuri classical dance
3. Mohiniyattam classical dance

Select the correct answer using the codes below.


A. 1 and 3 only
B. 2 only
C. 1, 2 and 3
D. None of the above

Your Answer : C
Correct Answer : C

Answer Justification :

Justification: S1: The word Kathak has been derived from the word Katha which means a story.
Kathakars or story-tellers, are people who narrate stories largely based on episodes from the epics,
myths and legends. It probably started as an oral tradition. Mime and gestures were perhaps added
later on to make the recitation more effective. Thus evolved a simple form of expressional dance,
providing the origins of what later developed into Kathak as we see it today.

The Vaishnavite cult which swept North India in the 15th century. and the resultant bhakti
movement contributed to a whole new range of lyrics and musical forms. The Radha-Krishna theme
Online Prelims TEST - 23 (TEXTBOOK)
( InsightsIAS Mock Test Series for UPSC Preliminary Exam 2020 )

proved immensely popular alongwith the works of Mirabai, Surdas, Nandadas and Krishnadas.

S2: The origin of Manipuri dance can be traced back to ancient times that go beyond recorded
history. The dance in Manipur is associated with rituals and traditional festivals, there are
legendary references to the dances of Shiva and Parvati and other gods and goddesses who created
the universe.

With the arrival of Vaishnavism in the 15th century A.D., new compositions based on episodes from
the life of Radha and Krishna were gradually introduced. It was in the reign of King Bhagyachandra
that the popular Rasleela dances of Manipur originated. It is said, that this 18th century philosopher
king conceived this complete dance form along with its unique costume and music in a dream.
Under successive rulers, new leelas, and rhythmic and melodic compositions were introduced.

S3: Mohiniyattam literally interpreted as the dance of ‘Mohini’, the celestial enchantress of the
Hindu mythology, is the classical solo dance form of Kerala.

According to a Puranic story, Lord Vishnu took on the guise of a ‘Mohini’ to seduce the Asuras, both
in connection with churning of the ocean and episode of the slaying of Bhasmasura.

Even Sattriya was influenced by Vaishnavism. We will cover more on this dance in the next test.

Q Source: http://ccrtindia.gov.in/kathak.php

68 In plants, which of the following parts are associated with gaseous exchange?
1. Roots
2. Leaves
3. Lenticels

Select the correct answer using the codes below.


A. 1 only
B. 1 and 2 only
C. 2 and 3 only
D. 1, 2 and 3

Your Answer : D
Correct Answer : D

Answer Justification :

Justification: Statement 3: Lenticels are small openings in the pits of the bark. The exchange of
gases takes place through the lenticels also, apart from the exchange through the openings in the
leaves.
Online Prelims TEST - 23 (TEXTBOOK)
( InsightsIAS Mock Test Series for UPSC Preliminary Exam 2020 )

S1 and S2: The roots take in air present in the soil. Leaves have tiny pores called stomata through
which they exchange gases. The breakdown of glucose in the plant cells is similar to that in other
living beings.

Q Source: Revision: UPSC based Q papers

69 Mission Purvodaya was launched by the government in order to


1. Develop India’s eastern region into an integrated steel hub
2. Improve the logistics and utilities infrastructure in Eastern India to help improve economic
prospects

Select the correct answer using the codes below.


A. 1 only
B. 2 only
C. Both 1 and 2
D. None of the above

Your Answer : C
Correct Answer : C

Answer Justification :

Justification: The Centre unveils Mission Purvodaya to develop eastern region into an integrated
steel hub.

The eastern belt has the potential to add over 75 percent of the country’s incremental steel capacity
envisioned by the National Steel Policy.

Through this programme, the government aims to transform logistics and utilities infrastructure
which would change the socio-economic landscape in the eastern India.

The steps, under mission, also include growth of steel industry along with employment
opportunities across the entire value chain.

The proposed Integrated Steel Hub, encompassing Odisha, Jharkhand, Chhattisgarh, West Bengal
Online Prelims TEST - 23 (TEXTBOOK)
( InsightsIAS Mock Test Series for UPSC Preliminary Exam 2020 )

and Northern Andhra Pradesh, would serve as a torchbearer for socio-economic growth of Eastern
India.

Q Source:
https://www.business-standard.com/article/news-ani/mission-purvodaya-will-drive-india-s-march-tow
ards-5-trillion-economy-says-dharmendra-pradhan-120011200033_1.html

70 Consider the following statements about the Transgender Persons (Protection of Rights) Act, 2019.
1. It states that a person will be recognized as transgender on the basis of a certificate of identity
issued by the concerned Zila Panchayat or Municipal Corporation, as the case may be.
2. The Act provides reservations to transgender persons for employment in government agencies or
government-aided agencies.
3. It allows organized begging with prior approval from the District Magistrate.

Select the correct answer using the codes below.


A. 1 and 3 only
B. 2 only
C. 1 only
D. None of the above

Your Answer : D
Correct Answer : D

Answer Justification :

Justification: The Social Justice Ministry notified the Transgender Persons (Protection of Rights)
Act, 2019.

According to the new definition, a transgender person is somebody “whose gender does not match
the gender assigned to that person at birth and includes trans-men or trans-women, persons with
intersex variations, gender-queers, and persons having socio-cultural identities such as kinnar,
hijras, aravani, and jogta”.

Statement 1: Highlights of the Bill:

The Act aims to stop discrimination against a transgender person in various sectors such as
education, employment, and healthcare. It also directs the central and state governments to
provide welfare schemes for them.

It states that a person will be recognized as transgender on the basis of a certificate of


identity issued by the District Magistrate. This certificate will be a proof of identity as
transgender and confer rights under this Bill.

Going by the Act, a person would have the right to choose to be identified as a man, woman or
transgender, irrespective of sex reassignment surgery and hormonal therapy.
Online Prelims TEST - 23 (TEXTBOOK)
( InsightsIAS Mock Test Series for UPSC Preliminary Exam 2020 )

It also requires transgender persons to go through a district magistrate and “district


screening committee” to get certified as a transperson.

Composition: The committee would comprise a medical officer, a psychologist or psychiatrist, a


district welfare officer, a government official, and a transgender person.

Statement 2 and 3: Criticisms:

The Act is silent on granting reservations to transgender persons.

It has prescribed punishments for organised begging. However, the Act doesn’t provide
anything to better to condition in those areas, it doesn’t provide for reservation.

It also does not mention any punishments for rape or sexual assault of transgender persons as
according to Sections 375 and 376 of the Indian Penal Code, rape is only when a man
forcefully enters a woman.

Q Source:
https://www.insightsonindia.com/2020/01/14/insights-daily-current-affairs-pib-summary-14-january-
2020/

71 The FAO-EU Forest Law Enforcement, Governance and Trade (FLEGT) Programme primarily seeks to

A. Reduce and eventually eliminate illegal logging


B. Increase forest cover in developing countries
C. Promote the movement of community forest ownership
D. Prevent frequent forest fires in tropical regions

Your Answer : A
Correct Answer : A

Answer Justification :

Learning: With the support of its donors, the FAO-EU FLEGT Programme funds projects created by
governments, civil society and private sector organizations in Latin America, Africa and Asia to
improve forest governance.

The Programme works in support of the European Commission’s Action Plan on FLEGT to promote
the legal production and consumption of timber by granting funds to projects, and assisting them at
all stages from the original design through to the final outcome.

Decreasing illegal logging contributes to the Sustainable Development Goals by alleviating poverty
(SDG 1), ensuring food security (SDG 2), mitigating climate change (SDG 13) and managing a
forests sustainably (SDG 15).
Online Prelims TEST - 23 (TEXTBOOK)
( InsightsIAS Mock Test Series for UPSC Preliminary Exam 2020 )

The Programme’s specific objective is to tackle illegal logging, promote trade in legal timber
products and ultimately contribute to sustainable forest management (SFM) and poverty reduction.

Q Source: http://www.fao.org/in-action/eu-fao-flegt-programme/background/objectives/en/

72 CGIAR (formerly the Consultative Group for International Agricultural Research) is


1. A Global partnership that unites organizations engaged in research for a food-secured future
2. Dedicated to do research in reducing rural poverty, increasing food security and improving human
health

Which of the above is/are correct?


A. 1 only
B. 2 only
C. Both 1 and 2
D. None of the above

Your Answer : C
Correct Answer : C

Answer Justification :

Justification: CGIAR research is dedicated to reducing rural poverty, increasing food security,
improving human health and nutrition, and ensuring sustainable management of natural resources.

It is carried out by 15 centers that are members of the CGIAR Consortium, in close collaboration
with hundreds of partners, including national and regional research institutes, civil society
organizations, academia, development organizations, and the private sector.

CGIAR research centers are generally run in partnership with other organizations, including
national and regional agricultural research institutes, civil society organizations, academia, and the
private sector.

These research centers are spread around the globe, with most centers located in the Global South,
at Vavilov Centers of agricultural crop genetic diversity.

CGIAR is unusual in that it is not part of an international political institution such as the United
Nations or the World Bank; it is an ad-hoc organization which receives funds from its members.

Q Source: Qs based on past year UPSC papers

https://www.cgiar.org/who-we-are/independent-science-and-partnership-council/

73 The Generalized System of Preferences (GSP) is often heard in the context of

A. Inter-bank loans for sovereign governments


B. Basel Accords
C. Global Trade rules
Online Prelims TEST - 23 (TEXTBOOK)
( InsightsIAS Mock Test Series for UPSC Preliminary Exam 2020 )

D. Monetary oversight

Your Answer : C
Correct Answer : C

Answer Justification :

Learning: It is a preferential tariff system which provides for a formal system of exemption from
the more general rules of the World Trade Organization (WTO).

Specifically, it is a system of exemptions from the most favoured nation principle (MFN) that
obliges WTO member countries to treat the imports of all other WTO member countries no
worse than they treat the imports of their "most favoured" trading partner.

In essence, MFN requires WTO member countries to treat imports coming from all other WTO
member countries equally, that is, by imposing equal tariffs on them.

GSP exempts WTO member countries from MFN for the purpose of lowering tariffs for the
least developed countries, without also lowering tariffs for rich countries.

For e.g. the European Union (EU) has amended its Generalised System of Preferences (GSP)
Scheme by introducing the concept of self-certification from 2017 onwards for which the office of
the DGFT has taken out Public Notice providing the details of the implementation of the EU GSP
self-certification scheme in India

Q Source: http://commerce.gov.in/InnerContent.aspx?Type=newsmenu&Id=341

https://en.wikipedia.org/wiki/Generalized_System_of_Preferences

74 Consider the following about the Indian Cyber Crime Coordination Centre.
1. It will be under the aegis of the Department of Electronics and Information Technology (DEITY).
2. It will coordinate all activities related to implementation of Mutual Legal Assistance Treaties
(MLAT) with other countries related to cybercrimes in consultation with the concerned nodal
authority.
3. The centre will assist in centralising cyber security investigations and bring together private
companies to contain cyber crime.

Select the correct answer using the codes below.


A. 1, 2 and 3
B. 2 and 3 only
C. 1 and 3 only
D. 2 only

Your Answer : A
Correct Answer : B
Online Prelims TEST - 23 (TEXTBOOK)
( InsightsIAS Mock Test Series for UPSC Preliminary Exam 2020 )

Answer Justification :

Justification: It will be set up under the newly created Cyber and Information Security (CIS)
division of the MHA.

It has seven components:

National Cyber Crime Threat Analytics Unit

National Cyber Crime Reporting Portal

National Cyber Crime Training Centre

Cyber Crime Ecosystem Management Unit

National Cyber Crime Research and Innovation Centre

National Cyber Crime Forensic Laboratory Ecosystem

Platform for Joint Cyber Crime Investigation Team.

Objectives:

To act as a nodal point in the fight against cybercrime

Identify the research problems/needs of LEAs and take up R&D activities in developing new
technologies and forensic tools in collaboration with academia / research institutes within
India and abroad

Suggest amendments, if required, in cyber laws to keep pace with fast changing technologies
and International cooperation

To coordinate all activities related to implementation of Mutual Legal Assistance Treaties


(MLAT) with other countries related to cybercrimes in consultation with the concerned nodal
authority in MHA.

Q Source:
https://www.insightsonindia.com/2020/01/11/insights-daily-current-affairs-pib-summary-11-january-
2020/
Online Prelims TEST - 23 (TEXTBOOK)
( InsightsIAS Mock Test Series for UPSC Preliminary Exam 2020 )

75 Raisina Dialogue is an annual geo-political event, organised by the Ministry of External Affairs and
Observer Research Foundation (ORF). Consider the following statements about it.
1. Participation is limited to Foreign and Defence Ministers and select think tanks from member
countries.
2. The dialogue is predicated on India’s vital role in the Indian Ocean Region and how India along with
its partners can build a stable regional and world order.

Select the correct answer using the codes below.


A. 1 only
B. 2 only
C. Both 1 and 2
D. None of the above

Your Answer : B
Correct Answer : B

Answer Justification :

Justification: The conference is a multi-stakeholder, cross-sectoral meeting involving policy and


decision-makers, including but not limited to Foreign, Defence and Finance Ministers of different
countries, high-level government officials and policy practitioners, leading personalities from
business and industry, and members of the strategic community, media and academia.

The Raisina Dialogue was born in 2016, in the belief that the Asian century that the world was
talking about was not about any exclusive geographical region. It was rather about the engagement
of global actors with Asia and of Asia with the world. So this dialogue took birth as a platform,
where the old and the new could work together, to discover their connections, their inter-
dependence.

It is designed to explore prospects and opportunities for Asian integration as well as Asia’s
integration with the larger world.

It is predicated on India’s vital role in the Indian Ocean Region and how India along with its
partners can build a stable regional and world order.

Q Source:
https://www.insightsonindia.com/2020/01/14/insights-daily-current-affairs-pib-summary-14-january-
2020/

76 Consider the following statements.


Strait of Hormuz waterway
1. Separates Iraq and Saudi Arabia
2. Links the Gulf of Oman to the Arabian Sea

Select the correct answer using the codes below.


A. 1 only
B. 2 only
C. Both 1 and 2
Online Prelims TEST - 23 (TEXTBOOK)
( InsightsIAS Mock Test Series for UPSC Preliminary Exam 2020 )

D. None of the above

Your Answer : D
Correct Answer : B

Answer Justification :

Justification: Strait of Hormuz is the waterway that separates Iran and Oman, linking the Gulf to
the Gulf of Oman and the Arabian Sea.

On the north coast lies Iran, and on the south are the UAE and Musandam, an enclave of Oman.

The Strait is 21 miles (33 km) wide at its narrowest point, but the shipping lane is just two miles
(three km) wide in either direction.

Why does Strait of Hormuz matter?

The Strait of Hormuz is one of the world’s most strategically important choke points.

Two-thirds of the oil and half the liquefied natural gas India imports come through the strait
between Iran and Oman.

As much as 18 million barrels of oil pass through the Strait of Hormuz every day, accounting for
one-third of the global oil trade. A third of the world’s LNG trade also passes through the strait.

Q Source:
https://www.insightsonindia.com/2020/01/14/insights-daily-current-affairs-pib-summary-14-january-
2020/

77 The WWF has identified 867 terrestrial ecoregions across the Earth's land surface, as well as
freshwater and marine ecoregions. The goal of this classification system is to ensure
1. that the full range of ecosystems will be represented in regional conservation and development
strategies
2. that the Global 200 selected among these ecoregions receive the most priority since these are
identified as critically endangered state

Which of the above is/are correct?


A. 1 only
B. 2 only
C. Both 1 and 2
D. None of the above

Your Answer : C
Correct Answer : A

Answer Justification :

Background: The WWF has identified 867 terrestrial ecoregions across the Earth's land surface, as
Online Prelims TEST - 23 (TEXTBOOK)
( InsightsIAS Mock Test Series for UPSC Preliminary Exam 2020 )

well as freshwater and marine ecoregions. The goal of this classification system is to ensure that the
full range of ecosystems will be represented in regional conservation and development strategies.

Of these ecoregions, the WWF selected the Global 200 as the ecoregions most crucial to the
conservation of global biodiversity. The Global 200 list actually contains 238 ecoregions, made up of
142 terrestrial, 53 freshwater, and 43 marine ecoregions.

These regions are priorities for conservation. According to WWF, an ecoregion is defined as a
"relatively large unit of land or water containing a characteristic set of natural communities that
share a large majority of their species dynamics, and environmental conditions".

The WWF assigns a conservation status to each ecoregion in the Global 200: critical or endangered;
vulnerable; and relatively stable or intact. Over half of the ecoregions in the Global 200 are rated
endangered.

Q Source: https://www.worldwildlife.org/publications/global-200

78 Consider the following statements.


1. Silappatikaram is a poetic rendition with details of Tamil culture and its varied religions.
2. Manimekalai, another tamil epic, is a sequel to Silappadikaram.

Which of the above is/are correct?


A. 1 only
B. 2 only
C. Both 1 and 2
D. None of the above

Your Answer : C
Correct Answer : C

Answer Justification :

Learning: These are two of the five epics of Tamil Literature that are considered in high regard by
the Tamil community.

The Silappatikaram is a poetic rendition with details of Tamil culture; its varied religions; its town
plans and city types; the mingling of different people; and the arts of dance and music. A poet-
prince from Kodungallur near Kochi (part of ancient Tamilakam, now in modern Kerala), referred to
by the pseudonym Ilango Adigal, is credited with this work.

The epic revolves around Kannagi, who having lost her husband to a miscarriage of justice at the
court of the Pandyan Dynasty, wreaks her revenge on his kingdom.

Manimekalai, by the poet Chithalai Chathanar, is a poem in 30 cantos. Its story is a sequel to
another of the Five Great Epics, Silappatikaram, and tells the story of the daughter of Kovalan and
Madhavi, who became a Buddhist Bikkuni.

Q Source: Regional Literature: Ancient India: Revision: Qs based on UPSC past year papers
Online Prelims TEST - 23 (TEXTBOOK)
( InsightsIAS Mock Test Series for UPSC Preliminary Exam 2020 )

79 Which of these are constitutional provisions with regard to minorities?


1. Freedom of Minority-managed educational institutions from discrimination in the matter of
receiving aid from the State
2. Right of ‘any section of the citizens’ to ‘conserve’ its ‘distinct language, script or culture
3. Provision for facilities for instruction in mother-tongue at primary stage

Select the correct answer using the codes below.


A. 3 only
B. 1 and 2 only
C. 1 and 3 only
D. 1, 2 and 3

Your Answer : D
Correct Answer : D

Answer Justification :

Justification: The Constitution provides two sets of rights of minorities which can be placed in
‘common domain’ and ‘separate domain’. The rights which fall in the ‘common domain’ are those
which are applicable to all the citizens of our country.

The rights which fall in the ‘separate domain’ are those which are applicable to the minorities only
and these are reserved to protect their identity. The distinction between ‘common domain’ and
‘separate domain’ and their combination have been well kept and protected in the Constitution.

The Minority Rights provided in the Constitution which fall in the category of ‘Separate Domain’ are
as under:-

right of ‘any section of the citizens’ to ‘conserve’ its ‘distinct language, script or culture’;
[Article 29(1)]

restriction on denial of admission to any citizen, to any educational institution maintained or


aided by the State, ‘on grounds only of religion, race, caste, language or any of them’; [Article
29(2)]

right of all Religious and Linguistic Minorities to establish and administer educational
institutions of their choice;[Article 30(1)]

freedom of Minority-managed educational institutions from discrimination in the matter of


receiving aid from the State;[Article30(2)]

special provision relating to the language spoken by a section of the population of any
State;[Article 347]
Online Prelims TEST - 23 (TEXTBOOK)
( InsightsIAS Mock Test Series for UPSC Preliminary Exam 2020 )

provision for facilities for instruction in mother-tongue at primary stage;[Article 350 A]

provision for a Special Officer for Linguistic Minorities and his duties; and [Article 350 B]

Sikh community’s right of ‘wearing and carrying of kirpans; [Explanation 1 below Article 25]

Q Source: http://ncm.nic.in/Constitutional_provisions.html

80 Consider the following statements.


1. Lai Haraoba is one of main festivals still performed in Manipur which has its roots in the pre-
Vaishnavite period.
2. Lai Haraoba is the earliest form of dance which forms the basis of all stylised dances in Manipur.

Select the correct answer using the codes below.


A. 1 only
B. 2 only
C. Both 1 and 2
D. None of the above

Your Answer : C
Correct Answer : C

Answer Justification :

Justification: The origin of Manipuri dance can be traced back to ancient times that go beyond
recorded history. The dance in Manipur is associated with rituals and traditional festivals, there are
legendary references to the dances of Shiva and Parvati and other gods and goddesses who created
the universe.

Lai Haraoba is one of the main festivals still performed in Manipur which has its roots in the pre-
Vaishnavite period. Lai Haraoba is the earliest form of dance which forms the basis of all stylised
dances in Manipur. Literally meaning - the merrymaking of the gods, it is performed as a
ceremonial offering of song and dance. The principal performers are the maibas and maibis (priests
and priestesses) who re-enact the theme of the creation of the world.

With the arrival of Vaishnavism in the 15th century A.D., new compositions based on episodes from
the life of Radha and Krishna were gradually introduced. It was in the reign of King Bhagyachandra
that the popular Rasleela dances of Manipur originated. It is said, that this 18th century philosopher
king conceived this complete dance form along with its unique costume and music in a dream.
Under successive rulers, new leelas, and rhythmic and melodic compositions were introduced.

Q Source: http://ccrtindia.gov.in/manipuri.php

81 Which of the following are migratory birds arriving in India?


1. Common Goldeneye
Online Prelims TEST - 23 (TEXTBOOK)
( InsightsIAS Mock Test Series for UPSC Preliminary Exam 2020 )

2. Blue Jay
3. Amur Falcon

Select the correct answer using the codes below.


A. 1 and 3 only
B. 1 and 2 only
C. 1, 2 and 3
D. 2 and 3 only

Your Answer : D
Correct Answer : A

Answer Justification :

Justification: You can find the full list here – it need not be remembered, but you may glance at it a
couple of times to become familiar with the names so that you can mark the correct answer in the
exam.

http://bnhsenvis.nic.in/Database/MigratorybirdsIndia_835.aspx

The number of migratory birds arriving in the country depends on various factors including habitat
quality and preference, human disturbances along their migratory route, pollution, hunting and
other biotic factors etc.

However, as per the latest ‘Asian Waterbird Census’ coordinated by the Wetlands International, the
population of threatened migratory birds in the entire flyway region which includes Central Siberia,
Mongolia, Central Asian republics, Iran, Afghanistan, Gulf States and the Indian sub-continent, are
either decreasing or stable.

The Government of India is providing financial and technical assistance to specialized institutions
like Salim Ali Centre for Ornithology and Natural History (SACON), Wildlife Institute of India (WII),
Bombay Natural History Society (BNHS) etc. for conducting various studies for monitoring
populations of important bird species and their habitat.

Q Source: General questions: Biodiversity

82 National Election Watch (NEW) is a/an

A. Autonomous body under the ECI


B. Non-partisan and non-governmental organization
C. A Wing of the UN Political Office
D. Informal inter-governmental organization

Your Answer :
Correct Answer : B

Answer Justification :
Online Prelims TEST - 23 (TEXTBOOK)
( InsightsIAS Mock Test Series for UPSC Preliminary Exam 2020 )

Justification & Learning: The Association for Democratic Reforms (ADR) is a non-partisan, non-
governmental organization which works in the area of electoral and political reforms.

Along with National Election Watch (NEW), which is a conglomeration of over 1200 organizations
across the country, ADR aims at bringing transparency and accountability in Indian politics and
reducing the influence of money and muscle power in elections.

Function & Research Areas

Election Watch or Analysis of Affidavits

Political Party Watch (PPW):

legal advocacy for Political and Electoral Reforms

Dissemination of information on elected representatives

Q Source: Often in news

83 Terrace Farming offers which of the following advantage(s) over conventional farming?
1. Increased water run-off
2. Help retain soil moisture
3. Cut down soil erosion

Select the correct answer using the codes below.


A. 1 and 2 only
B. 2 and 3 only
C. 3 only
D. 1 and 3 only

Your Answer : B
Correct Answer : B

Answer Justification :

Justification: Statement 1: Terrace farming is done on the steep slopes so that flat surfaces are
available to grow crops. They can reduce surface run-off and soil erosion.

Statement 2: The water break also allows more time for the water to settle into the soil.

Statement 3: It logically follows that as the velocity of water over the slopes is reduced, it lowers
soil erosion.

Q Source: ICSE Environmental Science: Xth


Online Prelims TEST - 23 (TEXTBOOK)
( InsightsIAS Mock Test Series for UPSC Preliminary Exam 2020 )

84 Consider the following ancient music treatises and their authors.


1. Brihaddesi: Matanga
2. Sangeeta Ratnakara: Haripala
3. Swaramelakalanidhi: Bharata

Select the correct answer using the codes below.


A. 2 only
B. 2 and 3 only
C. 1 only
D. 1 and 3 only

Your Answer :
Correct Answer : C

Answer Justification :

Justification: The history of the system of music that prevailed in India from ancient times, goes
back to the Vedas.

The earliest treatise we have on music is the Natya Sastra of Bharata. Other treatises on music after
Bharata, such as the Brihaddesi of Matanga, Sangeeta Ratnakara of Sharangadeva, Sangeet
Sudhakara of Haripala, Swaramelakalanidhi of Ramamatya, etc.,

The Tamil classic of the 2nd century A.D. titled the Silappadhikaram contains a vivid description of
the music of that period. The Tolkappiyam, Kalladam and the contributions of the Saivite and
Vaishnavite saints of the 7th and 8th centuries A.D. also serve as resource material for studying
musical history.

These provide us a fund of information about the different aspects of music and its development
during the different periods.

Q Source: http://ccrtindia.gov.in/carnaticclassicalmusic.php

85 The countries that will fall to the South of the line joining Kolkata and Kuala Lumpur are
1. Thailand
2. Vietnam
3. Indonesia

Select the correct answer using the codes below.


A. 1 only
B. 3 only
C. 1, 2 and 3
D. 1 and 3 only

Your Answer : B
Correct Answer : B

Answer Justification :
Online Prelims TEST - 23 (TEXTBOOK)
( InsightsIAS Mock Test Series for UPSC Preliminary Exam 2020 )

Q Source: Map-based Qs: Asia

86 The national parks that have Tropical Vegetation include


1. Simlipal National Park
2. Namdapha National Park
3. Khangchendzonga National Park

Select the correct answer using the codes below.


A. 1 and 2 only
B. 2 and 3 only
C. 3 only
D. 1 only

Your Answer : B
Correct Answer : A

Answer Justification :

Justification: Khangchendzonga and Nandadevi National park do not have tropical vegetation
because of the high altitude they are situated in. The vegetation there ranges from temperate to
boreal forests.

In contrast, Namdapha National Park hosts diverse vegetation with increasing altitude from tropical
moist forests to Montane forests, temperate forests and at the higher elevations, to Alpine meadows
and perennial snow.

Simlipal National Park largely has tropical deciduous vegetation.

Q Source: National parks of India

87 Forest fires can put which of the following adverse impacts on the environment?
Online Prelims TEST - 23 (TEXTBOOK)
( InsightsIAS Mock Test Series for UPSC Preliminary Exam 2020 )

1. Degradation of water catchment areas resulting in loss of water


2. Change in micro climate of the area by degrading living conditions

Which of the above is/are correct?


A. 1 only
B. 2 only
C. Both 1 and 2
D. None

Your Answer : B
Correct Answer : C

Answer Justification :

Justification: It can lead to:

Loss of valuable timber resources and depletion of carbon sinks

Degradation of water catchment areas resulting in loss of water

Loss Of biodiversity and extinction of plants and animals

Loss of wild life habitat and depletion of wild life

Loss of natural regeneration and reduction in forest cover and production

Global warming resulting in rising temperature

Loss of carbon sink resource and increase in percentage of CO2 in the atmosphere

Change in micro climate of the area making it unhealthy living conditions

Soil erosion affecting productivity of soils and production

Ozone layer depletion

Health problems leading to diseases Indirect effects on agricultural production: Loss of livelihood
for the tribals as approximately 65 million people are classified as tribals who directly depend upon
collection of non-timber forest products from the forest areas for their livelihood.

Q Source: ICSE Environment Class X


Online Prelims TEST - 23 (TEXTBOOK)
( InsightsIAS Mock Test Series for UPSC Preliminary Exam 2020 )

88 Match the following festivals with the states they are celebrated in:
1. Uttarayan: Gujarat
2. Maghi: Punjab
3. Saaji: Himachal Pradesh

Select the correct answer using the codes below.


A. 2 only
B. 1, 2 and 3
C. 1 and 3 only
D. 2 and 3 only

Your Answer : B
Correct Answer : B

Answer Justification :

Justification: Some of the festivals, other than the widely celebrated Makar Sankranti and Pongal,
are:

Magha Bihu: In Assam and many parts of the North East, the festival of Magha Bihu is
celebrated. It sees the first harvest of the season being offered to the gods along with prayers
for peace and prosperity.

Uttarayan: Gujarat celebrates it in the form of the convivial kite festival of Uttarayan.

Maghi: In Punjab, Makar Sankranti is celebrated as Maghi. Bathing in a river in the early
hours on Maghi is important.

Saaji: In Shimla District of Himachal Pradesh, Makara Sankranti is known as Magha Saaji.
Saaji is the Pahari word for Sankranti, start of the new month. Hence this day marks the start
of the month of Magha.

Kicheri: The festival is known as Kicheri in Uttar Pradesh and involves ritual bathing.

Q Source:
https://www.insightsonindia.com/2020/01/14/insights-daily-current-affairs-pib-summary-14-january-
2020/

89 The Brus can be found in

A. Assam, Mizoram and Tripura


B. Mizoram only
C. Arunachal Pradesh, Nagaland and Assam
Online Prelims TEST - 23 (TEXTBOOK)
( InsightsIAS Mock Test Series for UPSC Preliminary Exam 2020 )

D. Meghalaya and Sikkim

Your Answer : A
Correct Answer : A

Answer Justification :

Justification: The Brus--spread across Tripura, Mizoram and parts of southern Assam--are the most
populous tribe in Tripura. Also known as Reangs in the state, they are ethnically different from the
Mizos, with their own distinct language and dialect and form one of the 21 scheduled tribes of
Tripura.

Recently, the union home ministry finally inked the tripartite pact which now paves the way for
these displaced Bru tribals from Mizoram to permanently settle in Tripura.

The centre has signed a historic pact for permanent solution of Bru refugees’ issue.

The agreement is between Union Government, Governments of Tripura and Mizoram and Bru-
Reang representatives to end the 23-year old Bru-Reang refugee crisis.

This agreement will bring a permanent solution for the rehabilitation of thousands of Bru-Reang
people in Tripura. The government believes that this agreement will bring a bright future for them.
Bru-Reang people will be able to enjoy the benefits of all social-welfare schemes of governments.

Q Source:
https://www.insightsonindia.com/2020/01/17/insights-daily-current-affairs-pib-summary-17-january-
2020/

90 Consider the following statements about GSAT-30, recently seen in news.


1. The satellite will provide advanced telecommunication services to the Indian subcontinent.
2. It will replace all INSAT satellites currently deployed for space operations.

Select the correct answer using the codes below.


A. 1 only
B. 2 only
C. Both 1 and 2
D. None of the above

Your Answer : D
Correct Answer : A

Answer Justification :

Justification: The satellite will act as a replacement for the defunct INSAT-4A (not all INSAT
satellites). The satellite will provide advanced telecommunication services to the Indian
subcontinent. It will be used for VSAT networks, television uplinks, digital satellite news gathering,
DTH services and other communication systems. This is the 41st communication satellite launched
by ISRO and the 24th launch of ISRO satellite by Arianespace.
Online Prelims TEST - 23 (TEXTBOOK)
( InsightsIAS Mock Test Series for UPSC Preliminary Exam 2020 )

The satellite's main communication payload is 12 Ku band transponders for covering Indian
mainland and islands and 12 C-band transponders for extended coverage over Asia and Australia.

Launch Mass is about 3357 kg and Mission Life is more than 15 years. It was launched on Ariane-5
VA-251.

Q Source: https://www.isro.gov.in/Spacecraft/gsat-30

https://www.insightsonindia.com/2020/01/17/insights-daily-current-affairs-pib-summary-17-january-
2020/

91 Match the following folk traditions to their main feature.


1. Pankhida is sung by peasants of Assam during harvest.
2. The tradition of singing Rasiya Geet flourished in Braj.
3. In Pandavani, tales from Mahabharata are sung as a ballad and one or two episodes are chosen for
the performance.

Select the correct answer using the codes below.


A. 1 and 2 only
B. 2 and 3 only
C. 3 only
D. 1, 2 and 3

Your Answer : B
Correct Answer : B

Answer Justification :

Justification: Statement 1: Sung by the peasants of Rajasthan while doing work in the fields, the
peasants sing and speak even while playing algoza and manjira. The literal meaning of the word
‘Pankhida’ is lover.

Statement 2: The rich tradition of singing Rasiya Geet flourished in Braj which is the sacred land of
Lord Krishna’s charming leelas from time immemorial.

This is not confined to any particular festival, but is closely woven into the very fabric of daily life
and day to day chores of its people.

‘Rasiya’ word is derived from the word rasa (emotion) because rasiya means that which is filled
with rasa or emotion.

Statement 3: In Pandavani, tales from Mahabharata are sung as a ballad and one or two episodes
are chosen for the night’s performance. The main singer continuously sits throughout the
performance and with powerful singing and symbolic gestures he assumes all the characters of the
episode one after another.

Q Source: http://www.ccrtindia.gov.in/performingart.php
Online Prelims TEST - 23 (TEXTBOOK)
( InsightsIAS Mock Test Series for UPSC Preliminary Exam 2020 )

92 Consider the following about the functions of Animal Welfare Board of India.
1. To keep the law in force in India for the Prevention of Cruelty to Animals under constant study and
advise government appropriately
2. Encouraging by the grant of financial assistance for the establishment of animal sanctuaries.
3. Permitting the establishment of Biosphere reserve or Sanctuaries

Select the correct answer using the codes below.


A. 1 and 2 only
B. 3 only
C. 2 only
D. 1, 2 and 3

Your Answer : A
Correct Answer : A

Answer Justification :

Justification: It is a statutory advisory body on Animal Welfare Laws established in 1962 in


accordance with the provisions of the Prevention of Cruelty to Animals Acts 1960

Its functions are:

To keep the law in force in India for the Prevention of Cruelty to Animals under constant study
and to advise the government on the amendments to be undertaken in any such law from time
to time.

To take all such steps as the Board may think fit to ensure that unwanted animals are
destroyed by local authorities

To encourage by the grant of financial assistance or otherwise, the formation or establishment


of pinjarapoles, rescue homes, animals sanctuaries and the like, where animals and birds may
find a shelter

Statement 3: It is the National Board of Wildlife that generally gives such permissions.

Q Source: AR: ICSE 10th Environment Studies

93 Consider the following statements.


1. The Tappa consists of the song uttered in fast note patterns.
2. Thumri and Tappa do not require special training as the Dhrupad and Khyal forms of singing do.
3. A Thumri is usually set to ragas which signify the mood.

Which of the above is/are correct?


A. 1 and 2 only
Online Prelims TEST - 23 (TEXTBOOK)
( InsightsIAS Mock Test Series for UPSC Preliminary Exam 2020 )

B. 1 and 3 only
C. 2 and 3 only
D. 1, 2 and 3

Your Answer :
Correct Answer : B

Answer Justification :

Justification: The thumri is very lyrical in its structure and presentation. These forms are termed
as 'semi' or 'light' classical. Thumri is a love song and hence the textual beauty is very important.

This is closely coordinated with the musical rendition. And keeping in mind its mood a thumri is
usually set to ragas like Khamaj, Kaphi, Bhairavi and so on and the musical grammar is not strictly
adered to.

There are two styles of thumri singing: the Poorab or Banaras which is fairly slow and staid and the
Punjab style which is more mercurial. Rasoolan Devi, Siddheshwari Devi are prominent musicians of
this style.

S1: The Tappa consists of the song uttered in fast note patterns. It is a difficult composition and
needs much practice. Both the Thumri and Tappa require special training as do
the Dhrupad and Khyal forms of singing.

Q Source: http://www.ccrtindia.gov.in/performingart.php

94 Consider the following about South-East trade Winds.


1. They blow mainly in the Northern hemisphere near the equator.
2. The winds are deflected toward the East by the Coriolis Effect.

Which of the above is/are correct?


A. 1 only
B. 2 only
C. Both 1 and 2
D. None

Your Answer : B
Correct Answer : D

Answer Justification :

Justification: South-East trade Winds blow in the Southern Hemisphere. North-east trade winds
flow in Northern Hemisphere.

The surface air that flows from these subtropical high-pressure belts toward the Equator is
deflected toward the west in both hemispheres by the Coriolis effect.

These winds blow predominantly from the northeast in the Northern Hemisphere and from the
Online Prelims TEST - 23 (TEXTBOOK)
( InsightsIAS Mock Test Series for UPSC Preliminary Exam 2020 )

southeast in the Southern Hemisphere.

Because winds are named for the direction from which the wind is blowing, these winds are called
the northeasterly trade winds in the Northern Hemisphere and the southeasterly trade winds in the
Southern Hemisphere. The trade winds of both hemispheres meet at the doldrums.

Q Source: AR: Chapter 22: GC Leong Geography

95 Consider the following statements about some of the important nutrients and their role in plant
development.
1. Calcium is required for plant growth, cell division and enlargement.
2. Phosphorus (P) is an essential part of the enzymes which help the crop to fix light energy.
3. Nitrogen imparts dark green colour to plants.

Select the correct answer using the codes below.


A. 1 only
B. 2 and 3 only
C. 1 and 3 only
D. 1, 2 and 3

Your Answer : A
Correct Answer : D

Answer Justification :

Justification: Nitrogen (N) is an essential constituent of proteins and is present in many other
compounds of greatly physiological importance in plant metabolism
Online Prelims TEST - 23 (TEXTBOOK)
( InsightsIAS Mock Test Series for UPSC Preliminary Exam 2020 )

N is an integral part of chlorophyll, which is primary observer of light energy needed for
photosynthesis.

N also imparts vigorous vegetative growth and dark green colour to plants.

Calcium is required for plant growth, cell division and enlargement.

Phosphorus (P) is an essential part of the enzymes which help the crop to fix light energy.

Q Source: Revision: Past tests syllabus

96 Consider the following statements.


1. Sankirtana is a ritual singing, drumming and dancing that comes from Manipur.
2. Sankirtana is the only dance form from North-Eastern India that has been inscribed in the
Representative List of the Intangible Cultural Heritage of Humanity of UNESCO.
3. Sankirtana derives its inspiration from Pashupata cult of Shaivism.

Select the correct answer using the codes below.


A. 1 only
B. 1, 2 and 3
C. 1 and 2 only
D. 2 and 3 only

Your Answer : C
Correct Answer : C

Answer Justification :

Justification: Sankirtana, a ritual singing, drumming and dancing of Manipur encompasses an


array of arts performed to mark religious occasions and various stages in the life of the Vaishnava
people of the Manipur plains. Sankirtana practices centre on the temple, where performers narrate
the lives and deeds of Krishna through song and dance.

Inscription of Sankirtana on the Representative List will contribute to the visibility of Intangible
Cultural Heritage of India to the World while encouraging intercultural dialogue and subsequently
will promote respect for cultural diversity at all pervasive levels.

This is the list of all such heritages from India


http://www.unesco.org/culture/ich/en/lists?display=default&text=&inscription=0&country=00103&
multinational=3&type=0&domain=0&display1=inscriptionID#tabs

Q Source: http://www.ccrtindia.gov.in/performingart.php

97 Bio-fertilizers are useful in the conservation and management of soil in several ways. Which of these
microorganisms can serve as bio-fertilizers?
Online Prelims TEST - 23 (TEXTBOOK)
( InsightsIAS Mock Test Series for UPSC Preliminary Exam 2020 )

1. Anabaena cycadae
2. Cyanobacteria
3. Nostoc

Select the correct answer using the codes below.


A. 1 only
B. 2 only
C. 1 and 3 only
D. 1, 2 and 3

Your Answer : D
Correct Answer : D

Answer Justification :

Justification: Bio-fertilizers are specific types of living organisms like symbiotic bacteria,
Cyanobacteria (also called as blue green algae), sea weeds etc. that can bring about nutrient
enrichment of soil in many different ways. Anabaena azollae, Anabaena cycadae, Azolla pinnata and
Nostoc are different plants that enhance the productivity Of soil when added to it.

Bacteria like Rhizobium fix nitrogen for plants and Nostoc, Azolla, and Cyanobacteria that are great
nitrogen fixers, are used as bio- fertilizers in crop fields, most frequently.

Bio-fertilizers are useful in the Conservation and Management Of soil in following ways —

These are helpful in the replenishment and enhancement of soil fertility.

These fertilizers improve water holding capacity, aeration, porosity and drainage of soil.

These fertilizers are helpful in reclamation of unproductive soils.

These fertilizers are helpful in preventing soil erosion.

Q Source: ICSE Environment Class X

98 Consider the following statements about “Development Support Services to States for Infrastructure
Projects” (DSSS).
1. It is being implemented by the Ministry of Development of North Eastern Region.
2. DSSS is applicable only to states so far, and is yet to be piloted for Union Territories.

Select the correct answer using the codes below.


A. 1 only
B. 2 only
C. Both 1 and 2
Online Prelims TEST - 23 (TEXTBOOK)
( InsightsIAS Mock Test Series for UPSC Preliminary Exam 2020 )

D. None of the above

Your Answer :
Correct Answer : D

Answer Justification :

Justification: NITI Aayog and Ladakh Union Territory have signed a Memorandum of
Understanding, under which NITI Aayog will support the administration of the Ladakh through its
initiative “Development Support Services to States for Infrastructure Projects” (DSSS).

About Development Support Services For States/UTs (DSSS) For Infrastructure Projects:

Implemented by NITI Aayog.

Aim: To achieve transformational, sustained delivery of infrastructure projects with state of


art capacity disseminated at all levels of governance.

The key objective: Creating PPP success stories and rebooting infrastructure project delivery
models so a sustainable infrastructure creation cycle is established.

How it works? The DSSS Infrastructure initiative involves providing project level support from
Concept plan till financial closure to State Governments / UTs.

Q Source:
https://www.insightsonindia.com/2020/01/18/development-support-services-for-states-uts-dsss-for-in
frastructure-projects/

99 With reference to ‘Tolkappiyam’, consider the following statements.


1. It deals with the Tamil language grammar.
2. Its composition is sometimes ascribed to Sangam meetings by certain traditions.

Select the correct answer using the codes below.


A. 1 only
B. 2 only
C. Both 1 and 2
D. None of the above

Your Answer : C
Correct Answer : C

Answer Justification :

Justification: It deals with the Tamil language grammar and one of the oldest works in Tamil,
Online Prelims TEST - 23 (TEXTBOOK)
( InsightsIAS Mock Test Series for UPSC Preliminary Exam 2020 )

hence important.

Having three sections, the first two deal with language (linguistics) and the third about the subject
matters referred to in Tamil literature, the behavior of the Tamils.

The author of the work is Tolkappiyar (Tholkaappiyar). According to a fanciful Tamil tradition, this
literature was produced by poets of three “academies,” or sangams, that in the past were centred in
the southern Indian city of Madurai.

Q Source: http://www.ccrtindia.gov.in/performingart.php

http://www.thehindu.com/books/the-science-of-grammar/article3554215.ece

100 In anaerobic sludge digesters waste decomposing bacteria produce which of these gases?
1. Methane
2. Hydrogen sulphide
3. Carbon dioxide

Select the correct answer using the codes below.


A. 1 and 2 only
B. 2 and 3 only
C. 1 and 3 only
D. 1, 2 and 3

Your Answer : C
Correct Answer : D

Answer Justification :

Learning: Anaerobic digestion is particularly suited to organic material, and is commonly used for
industrial effluent, wastewater and sewage sludge treatment.

Anaerobic digestion, a simple process, can greatly reduce the amount of organic matter which
might otherwise be destined to be dumped at sea, dumped in landfills, or burnt in incinerators.

Using anaerobic digestion technologies can help to reduce the emission of greenhouse gases in a
number of key ways:

Replacement of fossil fuels

Reducing or eliminating the energy footprint of waste treatment plants

Reducing methane emission from landfills


Online Prelims TEST - 23 (TEXTBOOK)
( InsightsIAS Mock Test Series for UPSC Preliminary Exam 2020 )

Displacing industrially produced chemical fertilizers

Reducing vehicle movements

Reducing electrical grid transportation losses

Reducing usage of LP Gas for cooking

Q Source: Revision: UPSC based Q papers

You might also like